You are on page 1of 72

PRELIMS FITNESS – 2023

TEST 49 – ECONOMICS
EXPLANATION

1. Consider the following statement about Gender Budgeting in India:


1. It is a tool to achieve gender mainstreaming to ensure that the benefits of development reach
women as much as men.
2. The Union government publishes a Gender Budget Statement annually along with the Union
Budget.
3. Since 2005-06, the expenditure division of the Ministry of Finance has issued a note on gender
budgeting as a part of the Budget circular every year.
Which of the above statements is/are correct?
(a) 1 and 2 only
(b) 2 and 3 only
(c) 1 and 3 only
(d) 1, 2 and 3
EXPLANATION:
The Ministry of Women and Child Development (MoWCD) defines gender budgeting as a tool to
achieve gender mainstreaming so as to ensure that the benefits of development reach women as much
as men. So, statement 1 is correct.
The government publishes a Gender Budget Statement (GBS) every year along with the Union Budget.
GBS is a reporting mechanism for ministries or departments to review their programmes from a gender
perspective and present information on allocations for women. So, statement 2 is correct.
In 2005, the Secretary of the Planning Commission endorses the views of the Ministry of Finance and
advises “the Ministries/Departments to clearly bring out scheme-wise provisions and physical targets
in the annual plan proposals for 2005-06 and to carry out an incidence analysis of gender budgeting
from next financial year.”
Since 2005-06, the Expenditure Division of the Ministry of Finance issues a note on gender budgeting
as a part of the Budget Circular every year. Part A of the note reflects Women Specific Schemes, which
have 100 per cent allocation for women. Part B of the note reflects Pro-Women's Schemes, where at
least 30 per cent of the allocation is for women. So, statement 3 is correct.

2. Consider the following statements about the tools of Economic Stabilisation in India:
1. Fiscal policy is a policy under which the government uses taxation and public expenditure to
achieve various objectives of economic policy.
2. Monetary policy is a policy under which the Central Bank uses money supply and interest rates
to achieve macroeconomic objectives.
3. The increase in government expenditure has the potential to cause a crowding-out effect in the
economy

(3-F)
SIA-F-GS I
Which of the above statement is/are correct?
(a) 1 and 2 only
(b) 2 and 3 only
(c) 1 and 3 only
(d) 1, 2 and 3
EXPLANATION:
Fiscal policy is defined as the policy under which the government uses the instrument of taxation,
public spending and public borrowing to achieve various objectives of economic policy. Simply put, it
is the policy of government spending and taxation to achieve sustainable growth. So, statement 1 is
correct.
Monetary policy is the macroeconomic policy laid down by the central bank. It involves management
of money supply and interest rate and is the demand side economic policy used by the government of
a country to achieve macroeconomic objectives like inflation, consumption, growth and liquidity.
The RBI implements the monetary policy through open market operations, bank rate policy, reserve
system, credit control policy, moral persuasion and through many other instruments. So, statement
2 is correct.
A situation when increased interest rates lead to a reduction in private investment spending such that
it dampens the initial increase of total investment spending is called crowding out effect. Sometimes,
government adopts an expansionary fiscal policy stance and increases its spending to boost the
economic activity. This leads to an increase in interest rates. Increased interest rates affect private
investment decisions. A high magnitude of the crowding out effect may even lead to lesser income in
the economy. So, statement 3 is correct.

3. Consider the following statement about Tax:


1. It refers to a compulsory levy imposed by the government on individuals, groups of individuals, or
other legal entities.
2. It does not involve any definite and direct quid pro quo between the government and the taxpayer.
Which of the above statement is/are not correct?
(a) 1 only
(b) 2 only
(c) Both 1 and 2
(d) Neither 1 and 2
EXPLANATION:
A tax is a compulsory levy and those who are taxed have to pay the sums irrespective of any
corresponding return of services or goods by the government. In other words, a tax-payer does not
receive a definite and direct quid pro quo from the government. Note the word direct here. It is not a

(4-F)
SIA-F-GS I
price paid by the tax-payer for any definite service rendered or a commodity supplied by the
government.
The tax-payers do get many benefits from the government but no taxpayer has a right to any benefit
from the public expenditure on the ground that he is paying a tax. The benefits of public expenditure
may go to anyone irrespective of the taxes raid.
A tax is a liability imposed upon the tax assesses who may be individuals, groups of individuals, or
other legal entities. It is a liability to pay an amount on account of the fact that the tax assesses have
income of a minimum amount and from certain specified sources, or that they own certain tangible
or intangible property, or that they carry on certain economic activities which have been chosen for
taxation. Thus, a tax is a generalized exaction. It may be noted that a public receipt containing an
element of compulsion does not automatically become a tax. In order to be a tax, the absence of a
quid pro quo is necessary. As we shall see, the element of compulsion exists in case of some other
public receipts also. So, statements 1 and 2 are correct.

4. Consider the following statements about Fines and Taxes :


1. Fines are imposed to curb certain offenses, whereas taxes are imposed to generate revenue for the
State.
2. Unlike Tax, a Fine involves quid pro quo.
Which of the above statement is/are correct?
(a) 1 only
(b) 2 only
(c) Both 1 and 2
(d) Neither 1 and 2
EXPLANATION:
Fines (such as court fines) are also compulsory payments without any quid pro quo but they are
different from taxes because fines are imposed to curb certain offences and not to get revenue for the
State. In this sense, fines are not taxes. Similarly, import and export duties may be imposed with
different intentions in mind. If the intention is to get the revenue for the public treasury, they are
taxes. But if the intention is to regulate the flow of imports and exports, then they change their
character. So, statement 1 is correct.
Quid pro quo is a Latin term for "something for something" that originated in the middle ages in
Europe. It describes a situation when two parties engage in a mutual agreement to exchange goods or
services reciprocally.
As the object of a tax and fines are not to confer any special benefit upon any particular individual,
there is no element of 'quid pro quo' between the tax payer and the public authority. Both Tax and
fine do not involve quid pro quo. So, statement 2 is not correct.

(5-F)
SIA-F-GS I
5. Which of the following statements explains the term Tax Buoyancy ?
(a) It explains the relationship between the changes in the government's tax revenue growth
and the changes in GDP.
(b) It explains the relationship between the change in the government's tax revenue growth and the
changes in tax rates.
(c) It explains the legal description of the object with reference to which the Tax applies.
(d) It refers to the change in purchasing power to the changes in tax rates.
EXPLANATION:
Tax buoyancy is one of the key indicators to assess the efficiency of a government’s tax system.
Generally, as the economy achieves faster growth, the tax revenue of the government also goes up.
Tax buoyancy explains this relationship between the changes in government’s tax revenue growth and
the changes in GDP. In other words, it measures the responsiveness of tax mobilisation to economic
growth.
Tax buoyancy depends largely on:
➢ the size of the tax base
➢ the friendliness of the tax administration
➢ the reasonableness and simplicity of the tax rates
The highest tax buoyancy rate for the Union government during the last 28 years after economic
reforms was achieved in 2002-03. Tax buoyancy that year had risen to 2 at that time. This meant that
the Centre’s gross tax revenues had grown at double the rate at which the Indian economy had grown
in nominal terms. The 2014-19 period saw steady performance in tax buoyancy. So, option (a) is
correct.

6. Consider the following statements about the Cess and Surcharge:


1. The surcharge is a tax imposed on the Tax payable so as to make the tax structure more
progressive.
2. Cess can be imposed on both direct and indirect taxes.
3. Cess is a permanent tax collected by the government to develop a particular service or sector.
Which of the above statement is/are correct?
(a) 1 and 2 only
(b) 2 and 3 only
(c) 1 and 3 only
(d) 1, 2 and 3
EXPLANATION:
The surcharge is a tax imposed on the Tax payable so as to make the tax structure more progressive.
It is imposed on person who are earning higher income.
The term surcharge refers to an additional charge, fee, or tax that is added to the cost of a good or
service beyond the initially quoted price. A surcharge is often added to an existing tax and is not

(6-F)
SIA-F-GS I
included in the stated price of the good or service. The amount of a surcharge varies and can be a
fixed amount or a percentage of This charge may be imposed because of a governing body's need for
additional revenue or to defray the cost of increased commodity pricing. So, statement 1 is correct.
Cess is a form of tax charged/levied over and above the base tax liability of a taxpayer. A cess is
usually imposed additionally when the state or the central government looks to raise funds for specific
purposes. For example, the government levies an education cess to generate additional revenue for
funding primary, secondary, and higher education. Cess is not a permanent source of revenue for the
government, and it is discontinued when the purpose levying it is fulfilled. It can be levied on both
indirect and direct taxes. So, statement 2 is correct and statement 3 is not correct.

7. Which of the following taxes are subsumed by the introduction of the Goods and Service Tax (GST)?
1. Central Excise Duty
2. Service Tax
3. Basic Custom Duty
4. Counter Vailing Duty
5. Luxury Tax
Select the correct option using the code given below:
(a) 1, 2, 3, 4 and 5
(b) 1, 3, 4 and 5
(c) 1, 2, 4 and 5
(d) 2, 3, 4 and 5
EXPLANATION:
GST is a single tax on the supply of goods and services, right from the manufacturer to the consumer.
Credits of input taxes paid at each stage will be available in the subsequent stage of value addition,
which makes GST essentially a tax only on value addition at each stage. The final consumer will thus
bear only the GST charged by the last dealer in the supply chain, with set-off benefits at all the
previous stages.
1. At the Central level, the following taxes are being subsumed:
1. Central Excise Duty,
2. Additional Excise Duty,
3. Service Tax,
4. Additional Customs Duty commonly known as Countervailing Duty, and
5. Special Additional Duty of Customs.
2. At the State level, the following taxes are being subsumed:
1. Subsuming of State Value Added Tax/Sales Tax,
2. Entertainment Tax (other than the tax levied by the local bodies), Central Sales
Tax (levied by the Centre and collected by the States),
3. Octroi and Entry tax,

(7-F)
SIA-F-GS I
4. Purchase Tax,
5. Luxury tax, and
6. Taxes on lottery, betting and gambling.
So, option (c) is correct.

8. Which of the monetary policy is not likely to be used during the recession in the economy?
(a) Reduction in the interest rate of commercial banks.
(b) The Central Bank buy securities through open market operations.
(c) Reduction in CRR and SLR of the commercial banks.
(d) Increase in rate of Standing Deposit Facility.
EXPLANATION:
When economy is faced with recession or involuntary cyclical unemployment, which comes about due
to fall in aggregate demand, the central bank intervenes to cure such a situation. Central Bank takes
steps to expand the money supply in the economy and/or lower the rate of interest with a view to
increase the aggregate demand which will help in stimulating the economy.
The following three monetary policy measures are adopted as a part of an expansionary monetary
policy to measures are adopted as a part of an expansionary monetary policy to cure recession and to
establish the equilibrium of national income at full employment level of output.
➢ The central bank undertakes open market operations and buys securities in the open market.
Buying of securities by the central bank, from the public, chiefly from commercial banks will
lead to the increase in reserves of the banks or amount of currency with the general public.
With greater reserves, commercial banks can issue more credit to the investors and
businessmen for undertaking more investment. More private investment will cause aggregate
demand curve to shift upward. Thus buying of securities will have an expansionary effect.
➢ The Central Bank may lower the bank rate or what is also called discount rate, which is the
rate of interest charged by the central bank of country on its loans to commercial banks. At a
lower bank rate, the commercial banks will be induced to borrow more from the central bank
and will be able to issue more credit at the lower rate of interest to businessmen and investors
➢ The central bank may reduce the Cash Reserve Ratio (CRR) to be kept by the commercial
banks. In countries like India, this is a more effective and direct way of expanding credit and
increasing money supply in the economy by the central bank. Similar to the Cash Reserve
Ratio (CRR), in India there is another monetary instrument, namely Statutory Liquidity Ratio
(SLR) used by the Reserve Bank to change the lending capacity and therefore credit availability
in the economy.
The main purpose of SDF is to reduce the excess liquidity in the system, and control inflation. So,
during recession increase in rate of standing deposit facility is not a monetary policy used.
So, option (d) is correct.

(8-F)
SIA-F-GS I
9. Consider the following statements about Fiscal Deficit (FD):
1. It is the adverse fiscal balance which is a difference between the Revenue Receipts Plus Non-Debt
Capital receipts and the total expenditure.
2. It reflects the total borrowing requirement of the government.
Which of the above statement is/are correct?
(a) 1 only
(b) 2 only
(c) Both 1 and 2
(d) Neither 1 and 2
EXPLANATION:
Fiscal Deficit (FD) is the adverse fiscal balance which is a difference between the Revenue Receipts
Plus Non-Debt Capital Receipts (NDCR) i.e. total of the non-debt receipts and the total expenditure.
A country’s fiscal balance is measured by its government’s revenue vis-a-vis its expenditure in a given
financial year. Fiscal deficit, the condition when the expenditure of the government exceeds its revenue
in a year, is the difference between the two. Fiscal deficit is calculated both in absolute terms and as
a percentage of the country’s gross domestic product (GDP).
FD is reflective of the total borrowing requirement of Government. Revenue Deficit (RD) refers to the
excess of revenue expenditure over revenue receipts. Effective Revenue Deficit (ERD) is the difference
between Revenue Deficit and Grant-in-Aid for Creation of Capital Assets. Primary Deficit is measured
as Fiscal Deficit less interest payments. Effective Capital Expenditure (Eff-Capex) refers to the sum of
Capital Expenditure and Grants-in-Aid for Creation of Capital Assets. So, statements 1 and 2 are
correct.

10. Which of the following is the correct formula to calculate Effective Revenue Deficit (ERD)?
(a) Revenue deficit – non-debt creating capital assets.
(b) Total revenue expenditure – Total revenue receipts.
(c) Total Expenditure – Revenue Receipts + non-debt creating capital receipts
(d) Revenue deficit – Grant-in-aid for the creation of capital assets.
EXPLANATION:
The effective revenue deficit is the difference between the revenue deficit and grants for capital asset
creation. Every year, the centre gives grant-in-aid to the state and Union Territories, and with these
grants, they construct capital assets; however, this capital is not contributed to the central
government's capital expenditure. As a result, an effective revenue deficit has been created to measure
such expenditure. Revenue expenditures in the form of grants for capital asset creation are not
included in the Effective Revenue Deficit.
Effective Revenue Deficit = Revenue Deficit - Grants in aid for capital assets
During the 2011-12 Budget, the Rangarajan Committee on Public Expenditure developed the idea of
an effective revenue deficit in India. The ‘effective revenue deficit’ was introduced in Budget 2012

(9-F)
SIA-F-GS I
which excludes those revenue expenditures (or transfers) in the form of grants for the creation of
capital assets. Effective Revenue Deficit was established as a financial indicator in 2012-13.
So, option (d) is correct.

11. It refers to strengthening the government finances so that they are likely to continue in a robust way.
It is a process where the government's improving fiscal health is indicated by reducing the fiscal
deficit, which is manageable and bearable for the economy.
Which of the following concepts is referred to in the above-mentioned passage ?
(a) Fiscal Consolidation
(b) Fiscal Tightening
(c) Fiscal Expansion
(d) Budget Surplus
EXPLANATION:
Fiscal consolidation refers to strengthening the government finances so that they are likely to continue
in a robust way. Fiscal consolidation is a process where government’s improving fiscal health is
indicated by reduced fiscal deficit which is manageable and bearable for the economy. Improved
revenue realization and better aligned expenditure are thus components of fiscal consolidation.
The seeds for fiscal consolidation were sown in 1994 by the then Finance Minister Manmohan Singh.
In his budget speech for FY95, he highlighted the need for fiscal discipline and pronounced a policy
to end monetising the deficit. Till then the government was financing its deficit by creating money,
through unlimited recourse to the Reserve Bank, by issuing ad hoc treasury bills. As open market
borrowings piled up to fund the deficit, Yashwant Sinha in his budget speech for FY2001 called for a
strong institutional framework to ensure fiscal responsibility. This resulted in the enactment of the
‘Fiscal Responsibility and Budget Management (FRBM) Act, 2003’, which mandated limiting the fiscal
deficit to 3 per cent of GDP.
So, option (a) is correct.

12. Which of the following committee were appointed to review the implementation of the FRBM Act ?
(a) Kelkar committee
(b) NK Singh committee
(c) Narasimhan committee
(d) Basel committee
EXPLANATION:
The FRBM Review Committee headed by former Revenue Secretary, NK Singh was appointed by the
government to review the implementation of FRBM. In its report submitted in January 2017, titled,
‘The Committee in its Responsible Growth: A Debt and Fiscal Framework for 21st Century India’, the
Committee suggested that a rule based fiscal policy by limiting government debt, fiscal deficit and
revenue deficits to certain targets is good for fiscal consolidation in India. Main recommendations of
the NK Singh Committee are:

(10-F)
SIA-F-GS I
➢ Public debt to GDP ratio should be considered as a medium-term anchor for fiscal policy
in India. The combined debt-to-GDP ratio of the centre and states should be brought down to
60 per cent by 2023 (comprising of 40 per cent for the Centre and 20% for states) as against
the existing 49.4 per cent, and 21per cent respectively.
➢ Fiscal deficit as the operating target: The Committee advocated fiscal deficit as the operating
target to bring down public debt. For fiscal consolidation, the centre should reduce its fiscal
deficit from the current 3.5% (2017) to 2.5% by 2023.
➢ Revenue deficit target: The Committee also recommends that the central government should
reduce its revenue deficit steadily by 0.25 percentage (of GDP) points each year, to reach 0.8%
by 2023, from a projected value of 2.3% in 2017.
➢ Escape Clause to accommodate counter cyclical issues: the committee recommends fiscal
flexibilities to go above or below the fiscal deficit targets in the form of ‘escape clauses’. The
Committee set 0.5% as escape clause for fiscal deficit target.
So, option (b) is correct.

13. Which of the following is a Non-Debt Capital Receipt ?


(a) Market borrowings
(b) National Savings Certificates
(c) Recovery of loans
(d) Treasury Bills
EXPLANATION:
Revenue receipts comprise both tax and non-tax revenues, while capital receipts consist of capital
receipts and non-debt capital receipts. Non-debt capital receipts (NDCR) account for just 3 per cent
of the government’s total receipts.
The Union government usually lists non-debt capital receipts in two categories - recovery of loans,
and other receipts. Other receipts basically mean disinvestment proceeds from the sale of the
government’s share in public-sector companies.
There are two types of non-debt capital receipts, recoveries of loans and advances, and miscellaneous
capital receipts. The recoveries of loans and advances include recovery of loans and advances from
state governments and Union Territories with legislature, recovery of loans given to foreign
governments, recovery of loans and advances from PSUs and other autonomous bodies. So, option
(c) is correct.

14. Consider the following statements regarding the fiscal developments in India :
1. The fiscal deficit of the Union Government has been on an upward trend since 2019.
2. Direct taxes broadly constitute half of the Gross Tax Revenue.
3. The proportion of external liability in public debt FY22 is less than 5%.

(11-F)
SIA-F-GS I
Which of the above statement is/are correct ?
(a) 1 and 2 only
(b) 2 and 3 only
(c) 1 and 3 only
(d) 1, 2 and 3
EXPLANATION:
The fiscal deficit of the Union Government, which reached 9.2 per cent of GDP during the pandemic
year FY21, has moderated to 6.7 per cent of GDP in FY22 PA and is further budgeted to reach 6.4 per
cent of GDP in FY23. This gradual decline in the Union government's fiscal deficit as a per cent of
GDP, in line with the fiscal glide path envisioned by the government, is a result of careful fiscal
management supported by buoyant revenue collection over the last two years. So, statement 1 is
not correct.

Direct taxes, which broadly constitute half of the Gross Tax Revenue, have registered a growth of 26
per cent from April to November 2022, enabled by corporate and personal income tax growth. The
growth rates observed in the major direct taxes during the first eight months of FY23 were much
higher than their corresponding longer-term averages. So, statement 2 is correct.
BE FY23
Gross Tax Revenue 27.58
Direct taxes 14.20
a. Corporation Tax 7.20
b. Taxes on income other than 7.00
Corporation tax
The Union Government's total net liabilities in end-March 2021, 95.1 per cent were denominated in
domestic currency, while sovereign external debt constituted 4.9 per cent, implying low currency risk.
So, statement 3 is correct.

(12-F)
SIA-F-GS I
15. Which of the following statement refers to the Golden Rule of Public Finance ?
(a) The borrowing by the government should be used only for investment purposes and not to
meet excess current consumption expenditure except during times of recession.
(b) The government can withdraw money from the public treasury only after receiving prior approval
from the parliament.
(c) The government should always ensure to maintain an optimum level of taxation along with the
optimum level of expenditure
(d) The money withdrawn from the public treasury can never be used for other than sanctioned
purposes.
EXPLANATION:
The Golden Rule of Public Finance, namely, borrowing by the government should be used only for
investment purposes and not to meet excess current consumption expenditure except during times of
recession. In times of recession even government consumption expenditure will promote GDP growth.
the increase in public investment in infrastructure will also stimulate private investment.
The question of containing Government consumption expenditure, except during periods of recession
or economic slowdown, is important but reduction in fiscal deficit should not be elevated to a dogma.
A moderate amount of fiscal deficit and associated borrowing is good as long as it is used for increasing
public investment in physical infrastructure, education and health of the people. Even foreign
investment depends on our success in improving infrastructure. So, option (a) is correct.

16. Consider the following statements about the FRBM Act 2003:
1. It was enacted to institutionalize financial discipline, reduce India's fiscal deficit, and improve
macroeconomic management by moving towards a balanced budget.
2. The latest amendment to the FRBM Act set a target of a deficit of 3 percent of GDP by 2020-21.
3. The escape clause allows the Government to deviate from the target by 0.5% points in times of
exigencies, such as war or calamities of national proportion.
Which of the above statement is/are correct?
(a) 1 and 2 only
(b) 2 and 3 only
(c) 1 and 3 only
(d) 1, 2 and 3
EXPLANATION:
The Fiscal Responsibility and Budget Management Act, 2003 (FRBMA) was enacted to institutionalize
financial discipline, reduce India's fiscal deficit, and improve macroeconomic management and the
overall management of public funds by moving towards a balanced budget.
The main purpose was to eliminate the revenue deficit of the country and bring down the fiscal deficit
to a manageable 3% of the GDP by March 2008. However, due to the 2007 international financial
crisis, the deadlines for implementing the targets in the Act were initially postponed and subsequently
suspended in 2009. So, Statement 1 is correct.

(13-F)
SIA-F-GS I
A fiscal deficit is a difference between a government's expenditure and revenues when the former is
higher. The last amendment to the FRBM Act set a target of a deficit of 3 percent of GDP by 2020-21.
However, in the 2020 Union Budget, the target was relaxed to 3.5 percent as permitted under the Act.
The Centre used escape clauses to deviate from the fiscal consolidation roadmap.
In May 2016, the Government set up a committee under NK Singh to review the FRBM Act. The
Government believed the targets were too rigid. The committee recommended that the government
target a fiscal deficit of 3 percent of the GDP from March 31, 2020, cut it to 2.8 percent in 2020-21
and to 2.5 percent by 2023. So, Statement 2 is correct.
The FRBM Act mandates the Central Government to reduce its fiscal deficit to 3% of GDP. The initial
deadline to reach the 3% target was 2007-08, but it has been extended several times over the years.
In 2018, the deadline was again extended to 2020-21.
However, in the FY21 Budget, the target was relaxed to 3.5% as permitted under the FRBM Act. The
Centre made use of the escape clause to deviate from the fiscal consolidation roadmap. The Option
allows the Government to widen the deficit by 0.5 percentage points in times of exigencies, such as a
war or calamities of national proportion.
Because of higher expenditure and lower revenues on account of COVID-19, the number in FY21
came in at 9.3% of the GDP. The Government amended the FRBM Act in the Finance Bill last year to
enable the fiscal deficit beyond the permissible limit. For the ongoing fiscal, the Centre is targeting a
fiscal deficit of 6.8%, to be gradually brought down to below 4.5% in the next four years. So,
Statement 3 is correct.

17. Consider the following statements :


1. Population (1971)
2. Forest and Ecology
3. Tax effort
Which of the above is included as a criterion for devolution in the 15 th Finance Commission?
(a) 1 and 2 only
(b) 2 and 3 only
(c) 1 and 3 only
(d) 1, 2 and 3
EXPLANATION:
The Finance Commission is a constitutional body formed by the President of India to give suggestions
on center-state financial relations. The 15th Finance Commission is required to submit two reports.
The first report will consist of recommendations for the financial year 2020-21. The final report with
recommendations for the 2021-26 period will be submitted by October 30, 2020.
The 15th Finance Commission used the following criteria while determining the share of states:
➢ 45% for the income distance,
➢ 15% for the population in 2011(not Population (1971)
➢ 15% for the area,
➢ 10% for forest and ecology,
➢ 12.5% for demographic performance, and
➢ 2.5% for tax effort.

(14-F)
SIA-F-GS I
For 2020-21, the Commission has recommended a total devolution of Rs 8,55,176 crore to the states,
which is 41% of the divisible pool of taxes. This is 1% lower than the percentage recommended by the
14th Finance Commission. So, Option (b) is correct.

18. Consider the following criteria regarding the Central Public Sector Enterprises (CPSEs):
1. Having a Navratna Status.
2. Listed on the Indian stock exchange with minimum prescribed public shareholding under SEBI
regulations.
3. An average annual turnover of more than Rs. 5000 crores during the last 3 years.
4. An average net worth of more than Rs. 15,000 during the last 3 years.
Which of the following criteria are eligible to be considered for the grant of Maharatna status?
(a) 1, 2 and 3
(b) 1, 2 and 4
(c) 1, 3 and 4
(d) All the Above
EXPLANATION:
The criteria laid down by the Government for grant of Maharatna, Navratna and Miniratna status to
Central Public Sector Enterprises (CPSEs) are given below:
Criteria for grant of Maharatna status to CPSEs
The CPSEs meeting the following criteria are eligible to be considered for a grant of Maharatna status.
➢ Having Navratna status. So, Statement 1 is correct.

(15-F)
SIA-F-GS I
➢ Listed on the Indian stock exchange with minimum prescribed public shareholding under SEBI
regulations. So, Statement 2 is correct.
➢ An average annual turnover of more than Rs. 25,000 crores (not 5000 crores) during the last
3 years. So, Statement 3 is not correct.
➢ An average annual net worth of more than Rs. 15,000 crores during the last 3 years. So,
Statement 4 is correct.
➢ An average annual net profit after tax of more than Rs. 5,000 crores during the last 3 years
➢ Should have significant global presence/international operations.
So, Option (b) is correct.

19. Consider the following financial limits of the loans with respect to the Pradhan Mantri Mudra Yojana
(PMMY):
1. Shishu – covering loans upto 50,000
2. Tarun – Covering loans above 50,000 and upto 5 lakhs
3. Kishore – covering loans above 5 lakhs and upto 10 lakhs
How many pairs are correctly matched?
(a) One pair only
(b) Two pair only
(c) All the Above
(d) None of the Above
EXPLANATION:
Pradhan Mantri Mudra Yojana (PMMY) is a flagship scheme of the Government of India to "fund the
unfunded" by bringing such enterprises to the formal financial system and extending affordable credit
to them.
It enables a small borrower to borrow from all Public Sector Banks such as PSU Banks, Regional
Rural Banks and Cooperative Banks, Private Sector Banks, Foreign Banks, Micro Finance Institutions
(MFI) and Non-Banking Finance Companies (NBFC) for loans upto Rs 10 lakhs for non-farm income
generating activities. The Hon'ble Prime Minister launched the scheme on April 8, 2015.
Under the aegis of Pradhan Mantri MUDRA Yojana, MUDRA has already created the following
products/schemes.
➢ Shishu: covering loans up to 50,000/- So, Pair 1 is correct.
➢ Kishor: covering loans above 50,000/- and up to 5 lakhs So, Pair 2 is not correct.
➢ Tarun: covering loans above 5 lakhs and up to 10 lakhs. So, Pair 3 is not correct.
So, Option (a) is correct.

20. Which of the following influences Investments in the economy?


1. Savings Rate
2. Deflation
3. Market size and stability

(16-F)
SIA-F-GS I
Select the correct answer using the code given below:
(a) 1 and 2 only
(b) 2 and 3 only
(c) 1 and 3 only
(d) 1, 2 and 3
EXPLANATION:
Investment is expenditure on capital goods – for example, new machines, offices, and new technology.
Investment is a component of Aggregate Demand (AD) and also influences the capital stock and
productive capacity of the economy (long-run aggregate supply)
Investment levels are mainly influenced by the following:
➢ Saving rate
➢ The tax rate in the country
➢ Inflation (not deflation)
➢ The rate of interest in the banks
➢ Possible rate of return on capital
➢ Availability of other factors of production- cheap land, labor etc. and supporting infrastructure
– transports, energy and communication
➢ Market Size and stability.
So, Option (c) is correct.

21. Consider the following statements about the Infrastructure Investment Trusts (InvIT):
1. It is like a mutual fund, which enables direct investment of money from individual and
institutional investors in infrastructure projects to earn returns.
2. They are regulated by the SEBI.
Which of the above statement is/are NOT correct?
(a) 1 only
(b) 2 only
(c) Both 1 and 2
(d) Neither 1 and 2
EXPLANATION:
An Infrastructure Investment Trust (InvITs) is a Collective Investment Scheme similar to a mutual
fund, which enables direct investment of money from individual and institutional investors in
infrastructure projects to earn a small portion of the income as a return.
The InvIT is designed as a tiered structure with Sponsor setting up the InvIT, which in turn Invests
into the eligible infrastructure projects either directly or via special purpose vehicles (SPVs). So,
Statement 1 is correct.
The InvITs are regulated by the SEBI (Infrastructure Investment Trusts) Regulations, 2014. The
objective of InvITs is to facilitate investment in the infrastructure sector.

(17-F)
SIA-F-GS I
➢ InvITS are like mutual funds in structure. InvITs can be established as a trust and registered
with Sebi.
➢ An InvIT consists of four elements: 1) Trustee, 2) Sponsor(s), 3) Investment Manager and 4)
Project Manager. So, Statement 2 is correct.

22. Consider the following statements about the Black Box Model:
1. It is a type of investment model used by a company in testing hardware or programming on
hardware that belongs to another company.
2. The company uses this model without fully understanding the hardware with which they are
working.
3. This model also helps in analyzing human thought and behavior.
Which of the above statement is/are correct?
(a) 1 and 2 only
(b) 2 and 3 only
(c) 1 and 3 only
(d) 1, 2 and 3
EXPLANATION:
A black box model can also often be used by a company in testing hardware or programming on
hardware that belongs to another company. For example, a new video game may be developed on new
hardware that does not belong to the company developing the game. So, Statement 1 is correct.
The company will use a black box model to create the input going into the game and seeing the desired
output, but without fully understanding the hardware with which they are working. So, Statement
2 is correct.
A black box model is often used by behaviorists when considering human thought and behavior as
well. The human mind is considered a black box in this type of analysis, where a psychologist can say
things to a person, the input, and receive feedback from the person, the output. Behaviorists argue
that the actual cognitive process that occurs in the person is never observed and should not, therefore,
be considered when analyzing human behavior. So, Statement 3 is correct.

23. Which of the following entities release the Logistics Ease Across Different States (LEADS) report?
(a) Ministry of Commerce
(b) Ministry of Finance
(c) NITI Aayog
(d) Ministry of Road Transport & Highway
EXPLANATION:
The Government of India has prioritized developing a robust and cost-efficient logistics ecosystem. To
fulfill this vision, the logistics division of DPIIT, Ministry of Commerce & Industry, undertakes an

(18-F)
SIA-F-GS I
annual "Logistics Ease Across Different States (LEADS)" survey in all States/ UTs to assess and
suggest various improvements in the logistics sector of the country.
➢ The LEADS survey assesses the viewpoints of various users and stakeholders across the value
chain (Shippers, Terminal Infrastructure Service Providers, Logistics Service Providers,
Transporters and Government agencies) to understand the 'enabler' and 'impediments' to the
logistics ecosystem in the country. The annual survey processes the data received from
stakeholders (perception data) and States/ UTs (objective data) and ranks the logistics
ecosystem of each State/ UT using a statistical model.
➢ In the annual LEADS Report for 2021, Gujarat, Haryana and Punjab held the top three ranks
among States respectively, while Jammu Kashmir, Sikkim and Meghalaya were the top three
in North-eastern States and Himalayan UTs, respectively.
So, Option (a) is correct.

24. Which of the following committee was set up to oversee the revitalizing of the Public Private
Partnership?
(a) Vijay Kelalar committee
(b) Rangarajan committee
(c) Kaka Kalekar commission
(d) Urjit Patel committee
EXPLANATION:
Several infrastructure projects in India have been hit by various issues related to the Public-Private-
Partnership (PPP) model. In this context, the Vijay Kelkar panel recommended various measures to
revive the PPP model. The Union Finance Ministry appointed the panel in the Union Budget 2015-16.
Its recommendations are made available to the public.
Some of the major recommendations include the following:
➢ It recommended strengthening 3 main pillars of the PPP framework, viz. Governance,
Institutions and Capacity. The report endorsed the setting up a 3PI (a PPP institute of
excellence) for supporting institutional capacity-building activities.
➢ Independent sectoral regulators should be set up as and when a new sector is declared to
adopt the PPP model. The regulators should follow a unified approach. Without independent
regulators, the projects would be subjected to bureaucratic and political pressure.
➢ It should be explored to extend PPP into new sectors such as health, other social sectors, and
urban transport.
➢ The infrastructure PPP Project Review Committee (IPRC) should be set up for evaluating and
sending recommendations in a time-bound manner for stress in projects under the PPP model.
➢ An Infrastructure PPP Adjudication Tribunal (IPAT) should be set up, and the Chairperson will
constitute its benches as per the needs of the matter in question.
So, Option (a) is correct.

(19-F)
SIA-F-GS I
25. Which of the following five-year plan is based on Gadgil's strategy with the main focus on self-reliance?
(a) 3rd FYP
(b) 4th FYP
(c) 5th FYP
(d) 6th FYP
EXPLANATION:
Fourth Plan (1969-74) is based on the Gadgil Strategy, focusing mainly on growth with stability and
progress toward self-reliance. It sought to raise the standard of living through programs designed to
Promote equality and social justice. It laid particular emphasis on improving the conditions of the less
privileged and weaker sections of society, especially through the provision of employment and
education. So, Option (b) is correct.

26. Which of the following statements relates to the scheme Pradhan Mantri Suraksha Bima Yojana?
(a) It provides risk coverage for accidental death and complete or partial disability.
(b) It provides old-age income security through the provision of assured returns.
(c) It provides risk insurance to farmers against crop damage due to non-preventable risks.
(d) It provides health coverage for beneficiary families identified based on select deprivation and
occupational criteria.
EXPLANATION:
PMSBY is a one-year accidental insurance scheme renewable from year to year, offering coverage for
death or disability due to an accident. Individuals aged 18-70 years with a savings bank or a post
office account are entitled to enroll under the scheme.
➢ Under the scheme, risk coverage of ₹2 lahks for accidental death and complete disability and
₹1 lakh for partial disability is given to beneficiaries.
➢ And it achieved Since its inception, 31.3 crore beneficiaries have been enrolled under the
scheme, and 1.07 lakh claims have been disbursed as of November 30, 2022.
So, Option (a) is correct.

27. What are the possible effects if the Reserve Bank of India reduces the Repo Rate ?
1. Availability of cheaper loans to commercial banks.
2. It increases the money supply in the economy
Select the correct answer using the code given below :
(a) 1 only
(b) 2 only
(c) Both 1 and 2
(d) Neither 1 and 2

(20-F)
SIA-F-GS I
EXPLANATION:
The repo rate is the rate at which the central bank of a country (Reserve Bank of India) lends money
to commercial banks in the event of any shortfall of funds. The repo rate is used by monetary
authorities to control inflation.
⮚ The government increases the repo rate when they need to control prices and restrict
borrowing. On the other hand, the repo rate is decreased when there is a need to infuse more
money into the market and support economic growth. So, Statement 2 is correct.
⮚ Every time this Repo rate reduces, it means that other banks can now borrow money from RBI
at a much lower interest rate.
⮚ Commercial banks usually pass this benefit on to their customers by reducing the interest
rates on the loans they offer. Therefore, every time there is a cut in the repo rate, there usually
is a decline in the interest rates on loans offered by various banks. So, Statement 1 is correct.

28. Consider the following statements about the Neo-Banks :


1. They are FinTech companies that operate online without any physical branch networks.
2. Neo banks have no physical presence outside of their partnership with Traditional banks.
3. They do not have a banking license.
Which of the above statement is/are correct ?
(a) 1 and 2 only
(b) 2 and 3 only
(c) 1 and 3 only
(d) 1, 2 and 3
EXPLANATION:
Neo-banks are completely online-based digital banking platforms. They are financial technology
(FinTech) companies that operate online without physical branch networks. They offer money
management services digitally or via mobile apps. So, Statement 1 is correct.
Neobanks are similar to digital banks in terms of online operations. Digital banks are an extension of
established physical banks, while neo-banks have no physical presence outside their partnership with
traditional banks. They offer a comparatively limited set of services. However, these AI-driven services
come at a significantly lesser cost.
So, Statement 2 is correct
Neo banks allow customers to make deposits and withdraw money. They offer debit cards, investment
facilities, and more. They even provide credit and lending services.
However, most neo-banks do not have a banking license and cannot operate stand-alone; most neo-
banks partner with licensed banks to provide financial services. So, Statement 3 is correct.

(21-F)
SIA-F-GS I
29. Consider the following statement about the Ways and Means Advances (WMA) :
1. It is a borrowing facility available to tide over temporary mismatches in cash flows of the receipts
and expenditures of the Centre and States.
2. The Centre and States can use this facility to borrow funds from the RBI.
3. The interest charged is fixed at the RBI's reverse repo rate.
Which of the above statement is/are correct?
(a) 1 and 2
(b) 2 and 3
(c) 1 and 3
(d) 1, 2, 3
EXPLANATION:
The Ways and Means Advances (WMA) is a facility for both the Centre and states to borrow from the
RBI. These borrowings are meant to help them tide over temporary mismatches in cash flows of their
receipts and expenditures. In that sense, they aren't a source of finance per se. Section 17(5) of the
RBI Act, 1934 authorizes the central bank to lend to the Centre and state governments subject to
their being repayable "not later than three months from the date of the making of the advance."
The Reserve Bank of India (RBI) announced a 60% increase in the Ways and Means Advances (WMA)
limit of state governments over and above the level on March 31 to enable them "to undertake COVID-
19 containment and mitigation efforts" and "to better plan their market borrowings." So, Statements
1 and 2 are correct.
The interest rate on WMA is the RBI's repo rate, which is the rate at which it lends short-term money
to banks. That rate is currently 4.4%. However, governments are allowed to draw amounts in excess
of their WMA limits. The interest on such overdraft is 2 percentage points above the repo rate, now
6.4%. Further, no state can run an overdraft with the RBI over a certain period. So, Statement 3 is
not correct.

30. Which of the following adversely affects the internationalization of Indian currency ?
(a) Removal of restriction on trading domestic currency in the spot and forward market
(b) Use of Rupee in cross border transaction
(c) Restricting currency convertibility
(d) Encouraging Masala Bonds
EXPLANATION:
Internationalization of the Rupee is a process that involves increasing the use of the Indian currency
in cross-border transactions. It involves promoting the rupee for import and export trade and
other current account transactions, followed by its use in capital account transactions. The advantage
of Internationalization of the Rupee;

(22-F)
SIA-F-GS I
⮚ Using the rupee in cross-border transactions mitigates currency risk for Indian businesses.
Protection from currency volatility reduces the cost of doing business and enables better
business growth, improving the chances for Indian businesses to grow globally.
⮚ Removal of restriction on trading domestic currency in the spot and forward market.
⮚ Reducing dependence on foreign currency makes India less vulnerable to external shocks.
⮚ Masala Bonds are rupee-denominated bonds issued outside India by Indian entities.
Internationalization of the Rupee will encourage masala bonds.
Restricting currency convertibility adversely affects the internationalization of Indian currency. So,
option (c) is correct.

31. Consider the following statement about the DAKSH portal :


1. It is an initiative of the Reserve Bank of India.
2. It is a web-based end-to-end workflow application to monitor the compliance culture in supervised
entities like banks, NBFCs etc.
Which of the above statement is/are correct?
(a) 1 only
(b) 2 only
(c) Both 1 and 2
(d) Neither 1 and 2
EXPLANATION:
The Reserve Bank of India has taken various initiatives to strengthen supervision, including adopting
the latest data and analytical tools and leveraging technology to implement more efficient and
automated work processes. In continuation of this effort, Shri Shaktikanta Das, Governor of the RBI,
launched a new SupTech initiative named "(DAKSH) - Reserve Bank's Advanced Supervisory
Monitoring System," which is expected to make the Supervisory processes more robust.
⮚ (DAKSH) means 'efficient' & 'competent,' reflecting the underlying capabilities of the
application. (DAKSH)' is a web-based end-to-end workflow application through which RBI
shall monitor compliance requirements in a more focused manner to further improve the
compliance culture in Supervised Entities (SEs) like Banks, NBFCs, etc.
⮚ The application will also enable seamless communication, inspection planning and
execution, cyber incident reporting and analysis, provision of various MIS reports etc.,
through a Platform that enables anytime-anywhere secure access.
So, Both Statements are correct.

32. Consider the following statement about Inflation Indexed Bonds (IIB) :
1. It is a bond issued by RBI that guarantees a fixed yield regardless of the level of inflation in the
economy
2. IIB is eligible to be a part of the Statutory Liquidity Ratio.
3. Both principal and interest are protected against inflation.

(23-F)
SIA-F-GS I
Which of the above statement is/are correct?
(a) 1 and 2 only
(b) 2 and 3 only
(c) 1 and 3 only
(d) 1, 2 and 3
EXPLANATION:
Inflation Indexed Bonds are a type of bond that is designed to protect investors from the effects of
inflation. An inflation-indexed bond (IIB) is a bond issued by the Reserve Bank of India that guarantees
a fixed yield regardless of the level of inflation in the economy. The Government of India introduced
inflation-indexed bonds (IIB) in 2013 to lessen the attractiveness of gold as an investment and to lower
the Current Account Deficit. So, Statement 1 is correct
Inflation Indexed Bonds would be a G-Sec and issued as part of the approved Government market
borrowing program. Therefore, IIBs would automatically get SLR status. So, Statement 2 is correct
Inflation Indexed Bonds (IIBs) were issued in the name of Capital Indexed Bonds (CIBs) in 1997. The
CIBs issued in 1997 provided inflation protection only to the principal and not to interest payments.
The new product of IIBs, which was introduced in 2013, will provide inflation protection to both
principal and interest payments. So, Statement 3 is correct.

33. Consider the following statement about the Open Market Operation (OMO) of the RBI :
1. It refers to buying and selling of bonds issued by the Government by RBI in the open market.
2. RBI purchases Government securities to curb inflation.
Which of the above statement is/are correct ?
(a) 1 only
(b) 2 only
(c) Both 1 and 2
(d) Neither 1 and 2
EXPLANATION:
Open market operations are one of the tools the central bank (RBI) uses to affect the availability of
money and credit.
⮚ The term refers to a central bank buying or selling securities in the open market to influence
the money supply. So, Statement 1 is correct.
⮚ The RBI uses open market operations to manipulate interest rates, starting with the federal
funds rate used in interbank loans.
⮚ Buying securities adds money to the system, lowers rates, makes loans easier to obtain, and
increases economic activity.
⮚ Selling securities removes money from the system, raises rates, makes loans more expensive,
and decreases economic activity.
⮚ Thus, the RBI sells Government securities to curb inflation. So, Statement 2 is not correct

(24-F)
SIA-F-GS I
34. Consider the following statements about Vostro and Nostro accounts :
1. A vostro account is an account that a domestic bank holds for a foreign bank in the domestic
bank's currency.
2. A Nostro account is an account that a foreign bank holds for a domestic bank in the foreign bank's
currency.
Which of the above statement is/are not correct ?
(a) 1 only
(b) 2 only
(c) Both 1 and 2
(d) Neither 1 and 2
EXPLANATION:
A Vostro account is an account that a domestic bank holds for a foreign bank in the domestic bank's
currency, i.e., the rupee. The idea is to get around Western sanctions against Russia by carrying on
trade in rupees. The RBI allowed nine such accounts, including IndusInd Bank and Uco Bank.
In the case of trade with Russia, payments in rupee for the export and import of goods will go to these
Vostro accounts. The owners and beneficiaries of this money will be the exporters and importers in
both countries. The banks will keep a record of money transferred. So, Statement 1 is correct.
A Nostro account is an account held by a bank in another bank. It allows the customers to deposit
money in the bank's account in another bank. It is often used if a bank has no branches in a foreign
country.
Let's presume bank "A" does not have any branches in the US, but bank "B" does. Now, to receive the
deposits in the US, "A" will open a Nostro account with "B."
Now, if any customers in the US want to send money to "A," they can deposit it into A's account in
Bank "B." then, Bank "B" will transfer the money to Bank "A."
The main difference between deposit and Nostro accounts is that individual depositors hold the former
while foreign institutions hold the latter.
So, Statement 2 is correct.

35. Which of the following are the advantages of the internationalization of the Indian Rupee ?
1. It helps to mitigate currency risk for Indian businesses.
2. Reduce dependence on foreign currency.
3. Indian economy becomes less vulnerable to external shocks.
4. Reduce the need for holding foreign exchange reserves.
Select the correct answer using the code given below:
(a) 1, 2, 3
(b) 2, 3, 4
(c) 1, 3 and 4
(d) 1, 2, 3 and 4

(25-F)
SIA-F-GS I
EXPLANATION:
Internationalization of the Rupee is a process that involves increasing the use of the Indian currency
in cross-border transactions. It involves promoting the rupee for import and export trade and
other current account transactions, followed by its use in capital account transactions. The
advantage of Internationalization of the Rupee;
⮚ Using the rupee in cross-border transactions mitigates currency risk for Indian businesses.
Protection from currency volatility reduces the cost of doing business and enables better
business growth, improving the chances for Indian businesses to grow globally.
⮚ Removal of restriction on trading domestic currency in the spot and forward market
⮚ Reducing dependence on foreign currency makes India less vulnerable to external shocks
⮚ Masala Bonds are rupee-denominated bonds issued outside India by Indian entities.
Internationalization of the Rupee will encourage masala bonds.
⮚ Reduce the need for holding foreign exchange reserves.
⮚ Further, it could assist Indian exporters in getting advance payments in INR from overseas
clients and, in the long term, promote INR as an international currency once the rupee
settlement mechanism gains traction.
So, option (d) is correct

36. Which of the following committees were appointed with reference to Capital Account Convertibility ?
(a) Tarapore committee
(b) Narasimhan committee
(c) Justice Verma committee
(d) NK. Singh committee
EXPLANATION:
Capital account Convertibility (CAC) is the freedom to convert domestic currency into any other
foreign currency and vice-versa at a market-determined exchange rate for capital account-related
purposes. Capital account convertibility also makes it easier for domestic companies to tap the foreign
market.
Second Tarapore Committee on Capital A ~ Convertibility, 2006:
The fuller capital convertibility of the rupee seemed desirable at the end of 2006 when the committee
submitted its report. However, economic events, especially the global financial crisis of 2007-09,
brought about a sea change to the economic situation. The Indian economy would have been greatly
affected by the US global financial crisis if we had implemented the Tara pore committee
recommendations. We could not have coped with the extent of capital outflow that took place during
2008-09. Full CAC can pose a risk to financial stability without adequate safeguards, especially in
the current global environment. So, option (a) is correct.

(26-F)
SIA-F-GS I
37. Which of the following will likely cause capital flight in the Indian economy ?
1. Russia-Ukraine conflict
2. Monetary policy tightening by Federal Bank
3. Fall in price of US Treasury Bonds
Select the correct option using the code given below:
(a) 1 and 2 only
(b) 2 and 3 only
(c) 1 and 3 only
(d) 1, 2 and 3
EXPLANATION:
Capital flight is a large-scale exodus of financial assets and capital from a nation due to events such
as political or economic instability, currency devaluation or the imposition of capital controls. Foreign
portfolio investors (FPIs), which own around 19.5% of the market capitalization, have pulled out Rs
42,000 crore in June so far.
The possible reasons for capital flight:
⮚ The tightening of monetary policy by the US Fed
⮚ Rate hiking by other central banks, including in Britain and the Eurozone
⮚ An appreciating dollar
⮚ Rising inflation
⮚ The fall in the price of US Treasury Bonds gives a good yield. Thus, it will prompt foreign
portfolio investors to reduce their exposure to riskier assets in India (and other emerging
markets) and shift funds to safe-haven assets such as US Treasury bonds and gold.
⮚ Russia-Ukraine conflict (further worsened the current account deficit (CAD) outlook due to oil
price rises.)
So, option (d) is correct.

38. Which of the following is associated with the word 'Taper Tantrum' ?
(a) Monetary policy
(b) Fiscal Policy
(c) Forex Speculation
(d) Trade Strategy
EXPLANATION:
"Tapering," a phrase belonging to the world of monetary policy, refers to a central bank's strategy of
winding up a quantitative easing program used to purchase government debt and other assets to keep
borrowing costs low.
A taper tantrum is when investors have a "tantrum" or a reaction to news of the central bank slowing
or stopping bond purchases. Investors may react by selling bonds, which topples the price of bonds

(27-F)
SIA-F-GS I
and raises the yield. The sharp climb in bond yields after the central bank announcement is called a
taper tantrum.
⮚ In 2013, the US Federal Reserve decided to reduce the quantum of a bond-buying program
(quantitative easing). It began in response to the 2007-2009 global financial crisis and
recession that led to a sudden sell-off in global bonds and stocks.
⮚ As a result, many emerging market economies that received large capital inflows suffered
currency depreciation and capital outflows. Eventually, that phase came to be known as a
'taper tantrum.'
So, option (a) is correct.

39. What are the services available at white-label ATMs ?


1. Cash deposit
2. Bill payments
3. Request for cheque-book
4. Mini statement generation
5. Purchase of Re-load Vouchers for Mobiles
Select the correct answer using the code given below
(a) 1, 2 and 4 only
(b) 1, 2, 3 and 4
(c) 1, 2, 4 and 5 only
(d) 1, 2, 3, 4 and 5
EXPLANATION:
An ATM is a computerized machine that provides customers of banks the facility to access their
accounts for dispensing cash and to carry out other financial & non-financial transactions without
the need to visit the bank branch.
ATMs set up, owned and operated by non-banks are called WLAs. Non-bank ATM operators are
authorized under the Payment & Settlement Systems Act, 2007, by the Reserve Bank of India (RBI).
In addition to dispensing cash, ATMs / WLAs may offer many other services/facilities to customers.
Some of such services include:
➢ Account Information
➢ Cash Deposit
➢ Regular Bill Payment
➢ Purchase of Re-load Vouchers for Mobiles (not permitted at WLAs)
➢ Mini / Short Statement Generation
➢ PIN Change
➢ Request for ChequeBook
So, option (b) is correct.
Types of ATM In India

(28-F)
SIA-F-GS I
➢ Onsite ATM
These ATMs are inside the bank compound and hence are known as Onsite ATMs.
➢ Offsite ATMs
These ATMs are located in various places except inside the bank premises and are thus named Offsite
ATMs.
➢ Yellow Label ATM
These ATMs are mainly installed to provide for E-Commerce facilities.
➢ Brown Label ATM
These ATMs are not owned by the bank; instead, they are taken on lease to provide the service to the
customer.
➢ Orange Label ATM
These ATMs are used in the share transaction.
➢ Pink Label ATM
These ATMs are meant only for Women.
➢ Green Label ATM
These ATMs are installed for transactions related to agriculture.

40. Consider the following statements :


1. National Electronic Funds Transfer (NEFT) System is an electronic fund transfer system in which
the transactions received up to a particular time are processed in batches.
2. In the Real Time Gross Settlement, the transactions are processed continuously on a transaction-
by-transaction basis throughout the day.
3. The Real Time Gross Settlement is primarily meant for lesser value transactions with an upper
ceiling limit of ₹ 2,00,000/-
Which of the statements given above is/are correct ?
(a) 1 and 2 only
(b) 2 only
(c) 1, 2 and 3
(d) 2 and 3 only
EXPLANATION:
National Electronic Funds Transfer (NEFT) is a nationwide centralized payment system owned and
operated by the Reserve Bank of India (RBI). National Electronic Funds Transfer (NEFT) is an electronic
fund transfer system in which the transactions received up to a particular time are processed in
batches. Contrary to this, in RTGS, the transactions are processed continuously on a transaction-by-
transaction basis throughout the day. So, Statement 1 is correct.
Real Time Gross Settlement can be explained as a system where there is a continuous and real-time
settlement of fund transfers, individually on a transaction-by-transaction basis (without netting). 'Real
Time' means the processing of instructions at the time they are received; 'Gross Settlement' means

(29-F)
SIA-F-GS I
that the settlement of funds transfer instructions occurs individually. The NEFT system is available
round the clock throughout the year on all days, i.e., on a 24x7x365 basis. So, Statement 2 is
correct.
The RTGS system is primarily meant for large-value transactions. The minimum amount to be
remitted through RTGS is ₹ 2,00,000/- with no upper or maximum ceiling. So, Statement 3 is not
correct.

41. With reference to Service Area Approach, consider the following statements :
1. This scheme was introduced for the planned and orderly development of rural and semi-urban
areas and applied to all scheduled commercial banks, including Regional Rural Banks.
2. Under SAA, each bank branch in a rural or semi-urban area was designated to serve an area of
15 to 25 villages.
3. The RBI was responsible for meeting the needs of bank credit in its service area.
Which of the statements given above is/are correct ?
(a) 1 and 2 only
(b) 2 only
(c) 1, 2 and 3
(d) 2 and 3 only
EXPLANATION:
The Service Area Approach (SAA), introduced in April 1989 for the planned and orderly development
of rural and semi-urban areas, was applicable to all scheduled commercial banks, including Regional
Rural Banks.
The primary objective of SAA was to increase productive lending and forge effective linkages between
bank credit, production, productivity and increase in income levels. The SAA scheme was reviewed
from time to time, and appropriate changes were made in the scheme to make it more effective. So,
Statement 1 is correct.
Under SAA, each bank branch in a rural or semi-urban area was designated to serve an area of 15 to
25 villages, and the branch was responsible for meeting the needs of bank credit in its service area.
Under SAA, the allocation of villages among the rural and semi-urban branches of banks was made
not applicable for lending except under Government Sponsored Schemes. Thus, while commercial
banks and RRBs are free to lend in rural and semi-urban areas, the borrowers can approach any
branch for their credit requirements. So, Statement 2 is correct.
The branch (not the RBI) was responsible for meeting the needs of bank credit in its service area. It
was decided to dispense with the scheme's restrictive provisions while retaining the SAA's positive
features, such as credit planning and monitoring of credit purveyance. So, Statement 3 is not
correct

(30-F)
SIA-F-GS I
42. Consider the following statements about the Commercial Papers :
1. It is a short-term debt instrument companies issue to raise funds generally for up to one year.
2. They can be issued in denominations of Rs 5 lakh or multiples thereof.
3. Backed by collateral, they are usually sold at a discount to their face value.
Which of the statements given above is/are correct ?
(a) 1 and 2 only
(b) 2 only
(c) 2 and 3 only
(d) 1, 2 and 3
EXPLANATION:
Commercial paper, also called CP, is a short-term debt instrument issued by companies to raise funds
generally for a time period of up to one year. It is an unsecured money market instrument issued in
the form of a promissory note and was introduced in India for the first time in 1990.
Companies that enjoy high ratings from rating agencies often use CPs to diversify their sources of
short-term borrowings. This gives investors an additional instrument. Large banks or corporations
typically issue them to cover short-term receivables and meet short-term financial obligations, such
as funding for a new project. So, Statement 1 is correct.
CPs have a minimum maturity of seven days and a maximum of up to one year from the date of issue.
However, the instrument's maturity date should typically not go beyond the date up to which the
issuer's credit rating is valid. They can be issued in denominations of Rs 5 lakh or multiples thereof.
So, Statement 2 is correct.
Commercial Papers are not backed by collateral; only firms with high ratings from a recognized credit
rating agency can sell such commercial papers at a reasonable price. CPs are usually sold at a
discount to their face value. So, Statement 3 is not correct.

43. When there is a sudden surge in the Real GDP of an economy, it denotes
(a) Increase in the prevailing prices of goods and services
(b) An enormous increase in the export of Goods and services
(c) Attainment of the full employment potential of the country
(d) Increase in the volume of production of goods and services
EXPLANATION:
Real gross domestic product (GDP) is an inflation-adjusted measure that reflects the value of all goods
and services produced by an economy in a given year. Real GDP is expressed in base-year prices. It is
often called constant-price GDP, inflation-corrected GDP, or constant dollar GDP. But, real GDP
measures the total economic output of a country and is adjusted for price changes.

(31-F)
SIA-F-GS I
Real gross domestic product (GDP) is GDP given in constant prices and refers to the volume level of
GDP. Since these prices remain fixed, if the Real GDP changes, we can be sure that the production
volume is changing. So, option (d) is correct.

44. Which of the following is/are not considered part of Net National Income?
1. Any income earned by foreign residents within the country
2. Indian residents' income earned outside the country
3. Profits made on products manufactured in the country by foreign companies
Select the correct answer using the code given below:
(a) 1 and 2 only
(b) 2 and 3 only
(c) 1 and 3 only
(d) 1 only
EXPLANATION:
Net National Income is Gross National Income or Gross National Product less depreciation. Gross
National Product (GNP) is Gross Domestic Product (GDP) plus net factor income from abroad. It
measures the monetary value of all the finished goods and services produced by the country's factors
of production, irrespective of their location.
➢ Gross National Product (GNP) is the total value of all finished goods and services produced by
a country's citizens in a given financial year, irrespective of their location. So, Statement 2 is
correct.
➢ GNP signifies how a country's people contribute to its economy. It considers citizenship,
regardless of the location of the ownership. GNP does not include foreign residents' income
earned within the country. So, Statement 1 is not correct.
➢ GNP also does not count any income earned in India by foreign residents or businesses and
excludes products manufactured in the country by foreign companies. So, Statement 3 is
not correct.
➢ In calculation, GNP adds government expenditure, personal consumption expenditure, private
domestic investments, net exports, and income earned by nationals overseas, eliminating
foreign residents' income within the domestic economy. Moreover, GNP omits the value of
intermediary goods to avoid double counting, as these entries get included in the value of final
products and services.

45. With reference to Human Development Index, consider the following statements?
1. It reflects the status of poverty, inequality and human security in a country
2. India's HDI value is higher than South Asia's average human development.
Which of the statements given above is/are correct
(a) 1 only
(b) 2 only

(32-F)
SIA-F-GS I
(c) Both 1 and 2
(d) Neither 1 nor 2
EXPLANATION:
With an HDI value of 0.633, India ranked 132 out of 191 countries in the 2021 human development
index, according to a report released by the United Nations Development Programme (UNDP).
➢ The HDI measures progress on 3 key dimensions of human development - a long and healthy
life, access to education, and a decent standard of living.
➢ It is calculated using four indicators – life expectancy at birth, mean years of schooling,
expected years of schooling, and the Gross National Income (GNI) per capita.
➢ The HDI simplifies and captures only part of what human development entails. It does not
reflect inequalities, poverty, human security, empowerment, etc. So, Statement 1 is not
correct
India's HDI value continues to exceed South Asia's average human development. India's HDI value
has been steadily catching up to the world average since 1990 - indicating a faster than the global
rate of progress in human development. This is a result of policy choices made by the country over
time, including investments made in health and education.

So, Statement 2 is correct

46. With reference to Gini Coefficient Index, Consider the following statements
1. The Gini coefficient can be derived from both the Lorenz curve and Kuznets Curve
2. An increased value of the post-tax Gini Coefficient indicates a reduction in inequality among the
population
Which of the statements given above is/are correct ?
(a) 1 only
(b) 2 only
(c) Both 1 and 2
(d) Neither 1 nor 2

(33-F)
SIA-F-GS I
EXPLANATION:
Gini Coefficient Developed by Italian statistician Corrado Gini (1912), the Gini coefficient is the widely
used method to measure income and consumption inequalities. It is used to measure the degree of
concentration in the inequality of a variable in a distribution of its elements.
➢ With its values ranging between 0 and 1, it assumes a value of zero when there is perfect
equality in society. On the other hand, it takes the theoretical maximum of 1 when the income
level of every individual except one is zero in a population.
➢ The Gini coefficient thus gives a summary figure for the Lorenz curve. The ratio of the area
below the line of equality in the Lorenz curve to the total area of the triangle ABD in Figure 1.1
gives it.
➢ Gini Coefficient is derived from the Lorentz curve, which shows the relationship between
Population and National Income Source

So, Statement 1 is not correct


An increased value of post-tax Gini Coefficient indicates an increase in inequality among the
population. Economic experts generally consider a Gini figure below 0.40 to be within tolerable limits.
Chancel and Piketty’s report pegged India’s Gini coefficient at 0.41 to 0.49 for 2010. It is even likely
that it crossed 0.5, which is an alarming level of inequality. However, it is quite possible that the post-
tax Gini Coefficient for India is lower, as government welfare schemes are focused on the lower income
groups. The progressive rates that India uses for income tax slabs could also narrow the disparity. So,
Statement 2 is not correct.

47. With reference to the Index of Industrial Production(IIP), consider the following statements
1. IIP is released by the Office of Economic Adviser under the Ministry of Commerce and Industry
with 2011-12 as the base year
2. The classification of Industrial production under IIP is done based on the National Industrial
Classification of 2008
3. Under IIP, the electricity sector has less weightage than the mining sector
(34-F)
SIA-F-GS I
Which of the statements given above is/are correct
(a) 1 only
(b) 2 and 3 only
(c) 2 only
(d) 1 and 3 only
EXPLANATION:
The Central Statistics Office (CSO), Ministry of Statistics and Programme Implementation, revises the
base year of the macroeconomic indicators as a regular exercise to capture structural changes in the
economy and improve the quality and representativeness of the indices. In this direction, the base
year of the all-India Index of Industrial Production (IIP) has also been revised from 2004-05 to 2011-
12. Thus, the CSO releases the IIP, not the Office of Economic Adviser. So, Statement 1 is not
correct.
IIP in the revised series will continue representing the Mining, Manufacturing and Electricity sectors.
The revised series uses the National Industrial Classification (NIC) 2008 to classify industrial
production. As before, the unit coverage of IIP will cover entities in the organized sector units
registered under the Factories Act 1948. So, Statement 2 is correct.
The base year 2011-12 The base year 2004-05
Sector Weights Item Weights Item
(%) groups (%) groups
Mining 14.373 1 14.157 1
Manufacturing 77.633 405 75.527 397
Electricity 7.994 1 10.316 1
Total 100 407 100 399
Under IIP, the electricity sector has less weightage than the mining sector. So, Statement 3 is
correct.

48. Which among the following can be described as ‘demand deficient unemployment’.
(a) Disguised unemployment
(b) Cyclical unemployment
(c) Structural Unemployment
(d) Frictional Unemployment
EXPLANATION:
Cyclical unemployment: When there is an economy-wide decline in aggregate demand for goods and
services, employment declines, and unemployment increases. Therefore, it is sometimes referred to
as demand-deficient unemployment '. For instance, during the recent global slowdown, in late 2008,
many workers around the globe lost their jobs. So, option (b) is correct

(35-F)
SIA-F-GS I
Frictional unemployment: This type of unemployment refers to a transition period of looking for a new
job for different reasons, such as seeking a better job, being fired from a current job, or voluntarily
quitting a current job. The period of time between the current to a new job is referred to as frictional
or temporary unemployment. So, option (a) is not correct
Structural unemployment: This kind of unemployment occurs when there is a change in consumer
demand and technology in the economy. For instance, when computers were introduced, many
workers were dislodged because of a mismatch between the workers' existing skills and the job
requirement.
Although jobs were available, there was a demand for a new kind of skill and qualification. So, persons
with old skills did not get employment in the changed economic regime and remained unemployed.
This is called structural unemployment. So, option (c) is not correct
Disguised unemployment exists when part of the labor force is either left without work or is working
in a redundant manner such that worker productivity is essentially zero. It is unemployment that
does not affect aggregate output. An economy demonstrates disguised unemployment when low
productivity and too many workers are filling too few jobs. So, option (d) is not correct

49. With reference to the comparison of human and physical capital, consider the following statements
1. Physical capital is separable from its owner, whereas human capital is inseparable from its owner
2. Human capital does not depreciate with time, unlike physical capital
3. Human and Physical capital creates both private and Social benefits.
Which of the statements given above is/are correct?
(a) 1 only
(b) 2 and 3 only
(c) 1 and 3 only
(d) 1, 2 and 3
EXPLANATION:
Physical capital is referred to as one of the three main factors in the production process. Physical
capital consists of items like machinery, buildings, equipment, etc.
➢ Physical capital includes man-made goods that are used in the process of production for
converting raw materials to finished goods. Any new project requires a significant amount of
investment in physical capital.
➢ Physical capital is tangible and can be easily sold in the market like any other commodity.
➢ Physical capital is separable from its owner, whereas human capital is inseparable from its
owner.
➢ Human capital creates both private and social benefits, whereas physical capital creates only
private benefits. So, Statement 1 is correct.

(36-F)
SIA-F-GS I
Human Capital implies the knowledge that a worker brings to the company through education,
talents, abilities, knowledge, preferences, etc., that they have gathered over time. Consequently, the
employees are regarded as an asset whose value can be enhanced by spending on coaching and
improvement like any other firm asset.
➢ Human capital is intangible; it is endogenously built in the body and mind of its owner.
➢ Human capital is not sold in the market; only the services of the human capital are sold, and
hence, there arises the necessity for the owner of the human capital to be present in the place
of production.
➢ Human capital creates both private and social benefits, whereas physical capital creates only
private benefits. So, Statement 3 is not correct.
➢ Both forms of capital depreciate with time, but the nature of depreciation differs between the
two. Continuous use of machines leads to depreciation, and change in technology makes a
machine obsolete. In the case of human capital, depreciation takes place with ageing but can
be reduced, to a large extent, through continuous investment in education, health, etc. This
investment also facilitates the human capital to cope with change in technology which is not
the case with physical capital. So, Statement 2 is not correct.

50. Which among the following Survey is/are carried out by National Statistical Office (NSO)?
1. Household Consumer Expenditure Survey
2. Periodic Labour Force Survey
3. Consumer Price Index- Industrial Workers
Select the correct answer using the code given below:
(a) 1 and 2 only
(b) 2 only
(c) 1 and 3 only
(d) 3 only
EXPLANATION:
NSO is the central statistical agency of the Government-mandated under the Statistical Services Act
1980 under the Ministry of Statistics and Programme Implementation.
The Government of India will be conducting a new survey on Household Consumption Expenditure
Survey 2022-23 for a period of one year. The officials of the Field Operations Division of the National
Statistical Office will conduct the fieldwork for the same.
➢ Through this Survey, information will be collected to prepare budget shares of different
commodity groups in total consumption for the compilation of consumer price indices for rural
and urban India. In addition, statistical indicators of the level of living, social consumption
and well-being, and inequalities therein will also be compiled from the data collected in the
Survey.
So, Statement 1 is correct.

(37-F)
SIA-F-GS I
NSO launched the Periodic Labour Force Survey (PLFS) in April 2017 to estimate the key employment
and unemployment indicators (viz., Worker population ratio, labor force participation rate,
unemployment rate) in the short time interval of three months for the urban areas only in the ‘current
weekly status’ (CWS).
➢ The latest Periodic Labour Force Survey (PLFS) released by the National Statistical Office
(NSO) shows that the unemployment rate had shot up sharply during the nationwide
lockdown in 2020 during the first wave of the pandemic. So, Statement 2 is correct.
The Labour Bureau, an attached Ministry of Labour & Employment office, has been compiling
Consumer Price Index for Industrial Workers (CPI-IW) every month based on retail prices collected
from 317 markets spread over 88 industrially important centers in the country. The index is compiled
for 88 centers and All-India and is released on the last working day of the succeeding month. So,
Statement 3 is not correct.

51. With reference to the appointment of the RBI governor, consider the following statement
1. As per Constitutional provision, the RBI governor is appointed by the Prime Minister's Office on
the recommendation of the Union Finance Minister
2. The central board determines the salaries and allowances of the governor on the recommendation
of the Central Government
Which of the statements given above is/are correct?
(a) 1 only
(b) 2 only
(c) Both 1 and 2
(d) Neither 1 nor 2
EXPLANATION:
India’s central bank, i.e., the Reserve Bank of India (RBI) is a regulatory body responsible for the
regulation of the Indian banking system. It falls under the aegis of the Ministry of Finance,
Government of India. The RBI governor is appointed by the Prime Minister’s Office (PMO) on the
recommendation of the union finance minister as per Section 8(1)(a) of the Reserve Bank of India Act,
1934. There is no provision related to RBI in Constitution. So, Statement 1 is not correct.
The Reserve Bank's affairs are governed by a central board of directors. The board is appointed by the
Government of India in keeping with the Reserve Bank of India Act.
It includes Official Directors, including the RBI governor and not more than four Deputy Governors,
and non-official directors nominated by the government, including 10 Directors from various fields
and two government officials. Salaries and allowances of the governor and deputy governors may be
determined by the central board, with the approval of the central government. So, Statement 2 is
correct.

(38-F)
SIA-F-GS I
52. With reference to the Flexible Foreign Exchange Rate system, consider the following statement
1. Flexible exchange rate is determined based on the demand and supply of foreign currency
2. Under this system, the Central Bank of the country has no role in the regulation and management
of foreign currency
Which of the statements given above is/are correct?
(a) 1 only
(b) 2 only
(c) Both 1 and 2
(d) Neither 1 nor 2
EXPLANATION:
The foreign exchange rate is the price at which one currency is exchanged for another. It is the cost
incurred in local currency when purchasing a unit in a different currency.
In macroeconomics, a flexible foreign exchange rate system is a type of exchange rate system in which
market forces, such as supply and demand of money in the market, determine the exchange rate
between two currencies. The interactions of businesses, organizations, and other entities seeking to
purchase and sell foreign currency in the foreign exchange market influence the exchange rate. Like
a good, there is an inverse relation between the foreign exchange rate and its demand and positive
relation between the foreign exchange rate and its supply. So, Statement 1 is correct.
While operating with a flexible exchange rate, governments and central banks intervene in the foreign
exchange market to contain exchange rate volatility arising from surges and ebbs in capital flows. It
is only in a free-floating exchange rate system; governments and central banks do not participate in
the market for foreign exchange. The Reserve Bank issues licenses to banks and other institutions to
act as Authorised Dealers in the foreign exchange market. In keeping with the move towards
liberalization, the Reserve Bank has undertaken substantial elimination of licensing, quantitative
restrictions and other regulatory and discretionary controls. So, Statement 2 is not correct.

53. The Reserve Bank of India was set up on the basis of the recommendations of
(a) Simon Commission
(b) Lee Commission
(c) Butler commission
(d) Hilton Young commission
EXPLANATION:
The Reserve Bank of India was constituted to Regulate the issue of banknotes, maintain reserves to
secure monetary stability and operate the credit and currency system of the country to its advantage.
It was set up based on the recommendations of the Hilton Young Commission. The Hilton Young
Commission was a British commission appointed by Lord Reading, the viceroy in 1926, to review the
financial and economic situation of the British Empire. The Reserve Bank of India Act, 1934 (II of

(39-F)
SIA-F-GS I
1934) provides the statutory basis for the functioning of the Bank, which commenced operations on
April 1, 1935.
The Bank began its operations by taking over from the Government the functions performed by the
Controller of Currency and from the Imperial Bank of India, the management of Government accounts
and public debt. So, Option (d) is correct.

54. The Reserve Bank of India Act of 1934 permits the Reserve Bank to invest the reserves in which of
the following types of instruments?
1. Deposits with Bank for International Settlements
2. Deposits with foreign commercial banks
3. Sovereign-guaranteed debt instrument
Select the correct answer using the code given below:
(a) 1 and 2 only
(b) 2 and 3 only
(c) 1 and 3 only
(d) 1, 2 and 3
EXPLANATION:
The Reserve Bank of India Act of 1934 permits the Reserve Bank of India (RBI) to invest its reserves
in several types of instruments, including:
1. Government of India securities and treasury bills
2. Securities guaranteed by the central or state governments
3. Deposits with other central banks and the Bank for International Settlements (BIS)
4. Deposits with foreign commercial banks
5. Units of domestic mutual funds and exchange-traded funds (ETFs)
6. Sovereign-guaranteed debt instruments issued by foreign governments or supranational
entities
The investment policy aims to ensure safety, liquidity, and returns on the RBI's investments.
So, Option (d) is correct.

55. With reference to the National Strategy for Financial Inclusion, which among the following is/are the
Strategic objectives of Financial Inclusion in India?
1. Interest free lending to the weaker sections of the society
2. Access to livelihood and skill development
3. Financial Literacy and education for all the people
Select the correct answer using the code given below:
(a) 1 only
(b) 2 and 3 only
(c) 1 and 3 only
(d) 3 only

(40-F)
SIA-F-GS I
EXPLANATION:
The Reserve Bank of India (RBI) released the National Strategy for Financial Inclusion 2019-2024 on
January 10, 2020. It sets forth the vision and objectives of financial inclusion policies in India. The
report refers to financial inclusion as the process of ensuring access to financial services and timely
and adequate credit for vulnerable groups and low-income groups at an affordable cost.
Although Interest-free lending to weaker sections of society is a form of financial inclusion, it is not
the objective of RBI’s National Strategy for Financial Inclusion. So, Statement 1 is not correct.
RBI identified six strategic objectives of a national strategy for financial inclusion:
(i) universal access to financial services,
(ii) providing a basic bouquet of financial services,
(iii) access to livelihood and skill development, So, Statement 2 is correct.
(iv) Financial literacy and education, So, Statement 3 is correct.
(v) customer protection and grievance redressal, and
(vi) effective coordination.
To achieve this vision, it identified certain milestones such as:
(a) providing banking access to every village (or hamlet of 500 households in hilly areas) within a
five km radius by March 2020,
(b) strengthening digital financial services to create infrastructure to move towards a cashless
society by March 2022, and
(c) Ensure every adult has access to a financial service provider through a mobile device by March
2024.

56. With reference to NRI Deposit in India, Consider the following statements
1. NRI deposits are part of the capital account Balance in the balance of payment
2. The majority of NRI deposit is the under Non-Resident Ordinary Rupee Account
Which of the statements given above is/are correct?
(a) 1 only
(b) 2 only
(c) Both 1 and 2
(d) Neither 1 nor 2
EXPLANATION:
Non-Resident Indians (NRIs) live overseas and most of their earnings is in international currencies
such as the US dollar, Euro, and so on. As a result, NRIs might need an account that can hold and
convert their securities and money in Indian rupees. An NRI account serves this purpose. These
deposits can be repatriated by the NRI on maturity along with the interest earned.
From the balance of payments -the country's external sector balance sheet -perspective, NRI deposits
are capital flows and hence, vulnerable to outflows. They have been an important source of foreign

(41-F)
SIA-F-GS I
exchange at times of crisis. The capital account records the net change in foreign assets and liabilities
held by a country.

So, Statement 1 is correct.


The four types of NRI bank accounts most widely used and offered by the leading banks in India are:
1. Non-Resident External (NRE) Account
2. Non-Residency Ordinary (NRO) Account
3. Foreign Currency Non-Resident (FCNR) Account
4. Resident Foreign Currency (RFC) Account

Most NRI deposits are under Non-Resident External Deposits, not under Non-Resident Ordinary
Rupee Accounts. So, Statement 2 is not correct.

57. With reference to Special Drawing Rights(SDR), which of the statement given below is incorrect
(a) SDR is the non interest bearing international reserve asset
(b) The value of SDR is calculated based on five international currency
(c) SDR cannot be held by private entities and individuals
(d) SDR reduces the reliance on domestic or external debt for building reserves.
EXPLANATION:
The International Monetary Fund (IMF) is an international organization that was founded in 1944 to
promote international monetary cooperation, exchange rate stability, and economic growth. The IMF

(42-F)
SIA-F-GS I
is headquartered in Washington, D.C., and has 190 member countries. The Special Drawing Right
(SDR) is an interest-bearing international reserve asset created by the IMF in 1969 to supplement
other reserve assets of member countries. So, option (a) is not correct.
The SDR is based on a basket of five international currencies comprising the U.S. dollar, Japanese
yen, euro, pound sterling and Chinese Renminbi. It is not a currency nor a claim on the IMF, but is
potentially a claim on freely usable currencies of IMF members. The value of the SDR is set daily by
the IMF on the basis of fixed currency amounts of the currencies included in the SDR basket and the
daily market exchange rates between the currencies included in the SDR basket. The SDR also serves
as the unit of account of the IMF and some other international organizations, and financial obligations
may also be denominated in SDR. So, option (b) is correct.
SDRs can be held and used by member countries, the IMF, and certain designated official entities
called "prescribed holders,"—but it cannot be held, for example, by private entities or individuals. Its
status as a reserve asset derives from the commitments of members to hold and exchange SDRs and
accept the value of SDRs as determined by the Fund. So, option (c) is correct.
An SDR allocation is a way of supplementing Fund member countries’ foreign exchange reserves,
allowing members to reduce their reliance on more expensive domestic or external debt for building
reserves. So, option (d) is correct.
The IMF's Articles prescribe the conditions under which such allocations can be made, namely that
general allocations of SDRs should meet a long-term global need to supplement existing reserve assets
in a manner that will promote the attainment of the IMF's purposes and avoid economic stagnation
and deflation, as well as excess demand and inflation; and that these allocations should have the
broad support of SDR Department participants.

58. With reference to WTO's Appellate Body, consider the following statements:
1. It is a seven-member body with a fixed tenure of four years
2. The members are appointed by the dispute settlement body
3. The members of WTO Appellate Body are not eligible for reappointment.
Which of the statements given above is/are correct?
(a) 1 and 2 only
(b) 1 and 3 only
(c) 2 and 3 only
(d) 1, 2 and 3
EXPLANATION:
The World Trade Organization (WTO) is the only global international organization dealing with the
rules of trade between nations. At its heart are the WTO agreements, negotiated and signed by the
bulk of the world's trading nations and ratified in their parliaments. A dispute arises when a member
government believes another member government is violating an agreement or a commitment made

(43-F)
SIA-F-GS I
in the WTO. WTO's Appellate Body is a standing body of seven members to serve for four-year terms;
it hears appeals from reports issued by panels in disputes brought by WTO Members. So, Statement
1 is correct.
The General Council convenes as the Dispute Settlement Body (DSB) to deal with disputes between
WTO members. The DSB has authority to establish dispute settlement panels, refer matters to
arbitration, adopt panel, Appellate Body and arbitration reports, maintain surveillance over the
implementation of recommendations and rulings contained in such reports, and authorize suspension
of concessions in the event of non-compliance with those recommendations and rulings. The members
of the WTO's Appellate Body are appointed by the Dispute Settlement Body (DSB). So, Statement 2
is correct.
Each person may be reappointed for another four-year term in the WTO Appellate Body. Currently,
the Appellate Body is unable to review appeals given its ongoing vacancies. The term of the last sitting
Appellate Body member expired on 30 November 2020. One of the reasons for this is that the United
States has blocked the appointment/reappointment of members to the Appellate Body. The WTO
members have been trying to solve the current impasse, possibly reforming the body's functioning or
creating a new appellate mechanism. So, Statement 3 is not correct.

59. India has recently signed the "Economic Cooperation and Trade Agreement" with which the following
countries/organization
(a) UAE
(b) Gulf Cooperation Council
(c) MERCOSUR
(d) Australia
EXPLANATION:
IndAusECTA (Economic Cooperation and Trade Agreement) was signed last year, in April 2022; after
the Ratification and Exchange of Written Instruments, the Agreement came into force on
29th December 2022.
➢ It helps that Australia exports raw materials to India while India exports finished goods.
➢ The ECTA builds on this complementarity, creating win-win opportunities for the two
countries.
➢ The Department of Commerce has successfully operationalized two Trade Agreements this year
- India UAE FTA and Ind – Aus ECTA. The coming into force of Ind – Aus ECTA brings together
two major economies of the world - India, the 5th largest economy and Australia, the 14th
largest economy. Since the trade between the two countries is hugely complementary, this
offers opportunities on both sides and will pave the way for a win-win solution for both India
and Australia.
➢ India's imports from Australia are primarily (96%) raw materials & intermediate goods. They
are highly concentrated in Coal (74% of Australia's exports to India), of which 71.4% is Coal.

(44-F)
SIA-F-GS I
➢ On the other hand, India's exports to Australia are broad-based and dominated by finished
products (consumer goods). India also spends $ 4 bn approx—each year on the education of
students in Australia. So, option (d) is correct.

60. Consider the following statement


Statement 1: Balance of Trade is calculated on the basis of export and import of goods only
Statement 2: India's balance of trade has always been negative for the past four years
Which one of the following is correct in respect of the above statements?
(a) Both Statement 1 and Statement 2 are correct and Statement 2 is the correct explanation for
Statement 1
(b) Both Statement 1 and Statement 2 are correct, but Statement 2 is not the correct explanation for
Statement 1
(c) Statement 1 is correct, but Statement 2 is not correct
(d) Statement 1 is not correct, but Statement 2 is correct
EXPLANATION:
Current Account: The current account of the BOP can be broken into two parts—Viz., (a) balance of
trade and (b) balance of trade in services.
➢ Balance of Trade (BOT): The BOT deals only with exports and imports of merchandise (or
goods). The net balance in the BOT will show the monetary value of a country's difference in
exports and imports. So, statement 1 is correct.
➢ b) Balance of Trade in Services (BOS): The BOS shows net receipts on account of trade in
services (or what are also called invisible). We can broadly classify invisibly into five groups,
viz.,
(i) services, such as banking, insurance, shipping, civil aviation, royalty, consultancy
services, postal services, etc.
(ii) investment income, which includes profits and dividends on direct, portfolio and other
investments, and interest charges on bilateral and multilateral loans.
(iii) travel both business and tourist,
(iv) government transfers, and
(v) private transfers.
India's merchandise trade deficit has also been continuously falling since February 2020, and almost
after two decades, India experienced a trade surplus of USD 0.8 billion in June 2020. This is mainly
due to a huge fall in petroleum and crude oil imports, gold and silver, and exports recovery. Due to
COVID-19 Pandemic, India registered a trade surplus for a few months in FY 2021-22.
Thus, India's trade balance has not always been negative for the past four years. It saw positive growth
in the first quarter of 2020. So, statement 2 is not correct.

(45-F)
SIA-F-GS I
61. Which among the following measures can be taken to deal with the problem of the "Twin Deficit" in
India
1. Rationalizing non-capital expenditure
2. Boosting domestic production of goods and services
3. Reducing the import of crude oil and edible oil
Select the correct answer using the code given below
(a) 1 and 2 only
(b) 2 and 3 only
(c) 1 and 3 only
(d) 1, 2 and 3
EXPLANATION:
The current account deficit (CAD) and fiscal deficit comprise twin deficits that can impact the stock
market and investors.
➢ A fiscal deficit means higher expenditure over income. The gap between expenditure and
income is bridged through borrowing from the market.
➢ Current account deficit (CAD) is when the value of a country's imports of goods and services
is greater than its exports.
➢ The government needs to cut expenditures to keep the fiscal deficit under control. Since capital
expenditure should not be cut considering the growth requirement, a cut in revenue
expenditure is advisable.

(46-F)
SIA-F-GS I
➢ The Monthly Economic Review has also said that rationalizing non-capex expenditure has
become critical for protecting growth-supportive capex and avoiding fiscal slippages.
So, statement 1 is correct.
The government has a limited role in tackling the surging current account deficit. Still, it can formulate
a new strategy to boost exports by exploring newer markets, easing exporters' procedures and
releasing their tax dues in time.
➢ In order the lower the import bill, boosting domestic production of goods and services under
the Production Linked Incentives (PLI) schemes is vital.
➢ Reducing the import of crude oil and edible oil can tackle the Twin Deficit problem. So,
statements 2 and 3 are correct.

62. With reference to India's external sector, consider the following statements :
1. Remittances are the second largest major source of external financing in India
2. India is the largest recipient of remittances with anticipated remittance of US$ 100 billion.df
Which of the statements given above is/are correct ?
(a) 1 only
(b) 2 only
(c) Both 1 and 2
(d) Neither 1 nor 2
EXPLANATION:
Remittances are the second largest source of external financing after service export, contributing to
narrowing the Current Account Deficit and have always been a stable constituent of the BoP.
➢ According to the World Bank, India has the largest emigrant population and is the top
remittance recipient country with remittances anticipated to reach a milestone of US$100
billion in 2022.
➢ Remittances have benefitted from a gradual structural shift in Indian migrants' key
destinations from largely low-skilled, informal employment in the Gulf Cooperation Council
(GCC) countries to a dominant share of high-skilled jobs in high-income countries such as the
United States, the United Kingdom, and East Asia (Singapore, Japan, Australia, New Zealand).
So, statements 1 and 2 are correct.

(47-F)
SIA-F-GS I
63. With reference to India's major export destinations, arrange the following countries in descending
order :
1. USA
2. China
3. UAE
4. Netherland
Select the correct answer using the code given below :
(a) 1-2-3-4
(b) 2-1-4-3
(c) 1-3-4-2
(d) 2-1-3-4
EXPLANATION:
According to Economic Survey 2022-23, The USA remained the top export destination in April-
November, 2022, followed by UAE and the Netherlands. The Netherlands has displaced China from
the 3rd spot as India's exporting partner.
➢ India has diversified its export destinations over time. For example, the share of South Africa
in total exports has risen from 1.2 percent in FY19 to 2.0 percent in FY23 (April to November).
Over the same period, the share of Brazil has increased from 1.2 percent to 2.5 percent and
that of Saudi Arabia from 1.7 percent to 2.3 percent.
➢ As regards imports, China, UAE, USA, Russia, and Saudi Arabia have a joint share of 40
percent of the total imports of India. However, the share of China declined to 13.8 percent
during April-November 2022 from 15.5 percent a year ago. Similarly, the share of the USA
declined to 6.9 percent in April-November 2022 from 7.2 percent a year ago. So, option (c) is
correct.

64. With reference to Foreign investment, consider the following statements


1. Any investment through capital instrument by a person resident outside india in an unlisted
company is referred to as foreign Direct Investment(FDI)
2. Any investment by an investor in a listed company is referred to as Foreign Portfolio
Investment(FPI)
3. FDI Money is referred to as hot money as it is more stable than FPI
Which of the statements given above is/are correct ?
(a) 1 only
(b) 2 and 3 only
(c) 1 and 3 only
(d) 2 only

(48-F)
SIA-F-GS I
EXPLANATION:
Foreign Direct Investment (FDI) is the investment through capital instruments by a person resident
outside India (a) in an unlisted Indian company or (b) in 10 percent or more of the post-issue paid-up
equity capital on a fully diluted basis of a listed Indian company. So, statement 1 is correct.
Foreign Portfolio Investment is any investment made by a person resident outside India in capital
instruments where such investment is (a) less than 10 percent of the post-issue paid-up equity capital
on a fully diluted basis of a listed Indian company or (b) less than 10 percent of the paid-up value of
each series of capital instruments of a listed Indian company. So, statement 2 is not correct.
FDI and FPI are both important sources of funding for most economies. However, FDI is preferred by
most countries for attracting foreign investment since it is much more stable than FPI and signals
long-lasting commitment.
FPIs, on the other hand, have a higher degree of volatility because they tend to flee at the first signs
of economic trouble. These massive portfolio flows can exacerbate economic problems during periods
of uncertainty. FDI is a more stable long-term investment, FPI money is usually considered 'hot
money'. So, statement 3 is not correct.

65. Which among the following economic activities, Foreign Direct Investment is allowed in India ?
1. Real estate business
2. Tea plantation
3. Asset Reconstruction Companies
Select the correct answer using the code given below :
(a) 1 only
(b) 2 and 3 only
(c) 3 only
(d) None of the above
EXPLANATION:
Foreign investment in any form is prohibited in a company or a partnership firm or a proprietary
concern, or any entity, whether incorporated or not (such as Trusts), which is engaged or proposes to
engage in the following activities:
(i) Business of chit fund, or
(ii) Nidhi Company, or
(iii) Agricultural or plantation activities or
(iv) Real estate business or construction of farmhouses
(v) Trading in Transferable Development Rights (TDRs).
In addition to the above, investment in the form of FDI is also prohibited in certain sectors, such as:
(i) Retail Trading
(ii) Atomic Energy
(iii) Lottery Business

(49-F)
SIA-F-GS I
(iv) Gambling and Betting
(v) Agriculture (excluding Floriculture, Horticulture, Development of seeds, Animal Husbandry,
Pisciculture and Cultivation of vegetables, mushrooms etc., under controlled conditions and services
related to agro and allied sectors) and Plantations (Other than Tea plantations). Thus, foreign
investment in any form is prohibited in the real estate business. In Plantations, excluding Tea
plantations, FDI is prohibited in India. In an Asset reconstruction companies, FDI is allowed through
the government route. So, option (b) is correct.

66. "Draft Red Herring Prospectus (DRHP)," which was seen recently in the news, is related to :
(a) Imposing the countervailing duty
(b) Issuance of Initial Public Offering
(c) Resolution of a company under IBC
(d) Currency trade in the foreign exchange market
EXPLANATION:
When a company plans to raise money (for example: float an IPO) from the public by selling its shares
to investors, it files and submits a Draft Red Herring Prospectus (DRHP), also known as an 'offer
document' or 'preliminary registration document,' with the market regulator SEBI (Securities and
Exchange Board of India).
➢ DRHP, a preliminary legal prospectus, is a crucial communication link between the company
and its investors and stakeholders.
➢ The government plans to dilute its stake in LIC through the Initial Public Offering (IPO). LIC,
in February 2022, had filed the Draft Red Herring Prospectus (DRHP) before the markets
regulator SEBI for the IPO. SEBI approved the draft papers, and the insurer is in the process
of filing a request for a proposal with changes.
So, option (b) is correct.

67. With reference to the estimation of National Income, which among the following is/considered as the
Stock variables ?
1. National Income
2. Population
3. Money supply
4. Wealth
Select the correct answer using the code given below:
(a) 1,2 and 3 only
(b) 1 and 3 only
(c) 2,3 and 4 only
(d) 2 and 4 only

(50-F)
SIA-F-GS I
EXPLANATION:
The distinction between stock variables and flow variables is very significant for national income
estimates.
➢ Stock variables:
• A stock is a quantity that is measured at a point in time, i.e., at 4 p.m. on 31 March
etc., wealth, population, money supply etc., are stock concepts.
• It has no time dimension. It is a stock of capital.
• Like a balance sheet, a stock references a particular date on which it shows the stock
position. Clearly, a stock has no time dimension (length of time) as against a flow that
has a time dimension.
• Whereas, National income is a flow concept as it is measured over a period of time,
usually a year. So, Option (c) is correct.
➢ Flow variables:
A flow is a quantity measured over time, i.e., days, months, years, etc. It has a time dimension.
National income and population growth are flow concepts. It has a time dimension. National
income is a flow. It describes and measures the flow of goods and services which become
available to a country during a year. So, statement 1 is not correct.

68. Which among the following best describes the "Effective Rate of protection" ?
(a) Change in value added in the domestic industry due to tariff protection.
(b) Change in the total tax revenue due to change in the custom duty
(c) Change in the foreign exchange reserve due to a change in the value of a currency
(d) Change in the volume of external trade due to change in domestic policy measures
EXPLANATION:
The Effective Rate of protection (ERP) measures the extent of protection granted to the domestic
industry by the given structure of nominal tariff rates. It measures the change in value added in the
domestic industry due to tariff protection.
➢ Remember, value added is the difference between the price of the 'final good produced by an
industry, less the cost of inputs required per unit of production of the final good.
➢ The 'effective rate of protection' is also used to measure the extent of actual protection granted
to value added in a particular industry by the entire tariff structure (covering final and
intermediate goods).
➢ The domestic value added equals the price of the final commodity minus the cost of the
imported inputs going into the production of the commodity. When no or lower tariffs are
applied on imported inputs than on the final imported product, the Rate of protection, called
the effective Rate of production, exceeds the nominal tariff rate. So, Option (a) is correct.

(51-F)
SIA-F-GS I
69. Which among the following can be considered as Non-Tariff Barriers
1. Imposing restrictions on the quantum of imports
2. Stipulation of Local content requirement in the production of final goods
3. Levying special duty on a foreign company that sells goods below the cost of production
Select the correct answer using the code given below
(a) 1 and 2 only
(b) 2 and 3 only
(c) 1 and 3 only
(d) 1, 2 and 3
EXPLANATION:
Non-Tariff Barriers (NTBs) impose direct restrictions on the inflow of imported goods. For instance,
conventional Non-Tariff Barriers (NTBs), like import quotas, directly restrict the quantum of imports
into the domestic country. So, Statement 1 is correct.
The following are some of the other important NTBs commonly used by countries following
protectionist policies:
➢ Exchange controls: Exchange controls are restrictions imposed by countries' Central Banks
that directly limit domestic residents' ability to acquire foreign currency in exchange for
domestic currency.
➢ Import deposit scheme
➢ Health and safety standards
➢ Customs valuation procedure: Under this procedure, the importing country would artificially
enhance the value of the imported goods under some pretext, which would raise the duty on
it under a system of ad valorem tariffs
➢ Local content requirements: This is a practice followed especially in developing countries,
which requires that some stipulated portion of a final good be produced domestically. Or it
may be stated that a certain specified fraction of the final goods price must represent domestic
value added. The underlying logic is to promote the local production of certain intermediate
goods. From the importing finns viewpoint, there is no restriction on imports. They can import
more as long as they buy more from local firms. From the domestic intermediate industries'
point of view, a local content requirement provides trade protection in the same way as an
import quota. So, Statement 2 is correct.
Anti-dumping duty is a tariff imposed on imports manufactured in foreign countries that are priced
below the fair market value of similar goods in the domestic market.
➢ Anti-dumping duty is imposed to protect local businesses and markets from unfair competition
by foreign imports.

(52-F)
SIA-F-GS I
➢ Thus, Levying special duty on a foreign company that sells goods below the cost of production
is an anti-dumping dump, which is tariff Barriers. So, Statement 3 is not correct.

70. Which among the following best describes the term 'Tariffication.'
(a) Bringing all goods and services under the WTO Tariff regime
(b) Conversion of Non-tariff barriers into tariff barriers
(c) Protection of developing and least developed nations with differential tariff regime
(d) Conversion of tariff barriers into Non-tariff barriers
EXPLANATION:
Tariffication is a process where non-tariff barriers such as quotas and voluntary export restraints are
converted into tariff equivalents in import quantities, that is, tariffs that are set to result in the original
trade volume.
➢ This process makes it easier to assess true levels of protectionism, creates a solid base for
further negotiations and subsequent tariff reductions, and is thus in accordance with GATT
(WTO) principles.
➢ Its Procedures are related to the agricultural market-access provision in which all non-tariff
measures are converted into tariffs. So, Option (b) is correct.

71. With reference to WTO Subsidies Committee, consider the following statements
1. The committee was established based on Agreement on Agriculture to maintain the list of
subsidies allowed by WTO members
2. The committee is represented by all member countries of the WTO
Which of the statements given above is/are correct
(a) 1 only
(b) 2 only
(c) Both 1 and 2
(d) Neither 1 nor 2
EXPLANATION:
The Agreement on Subsidies and Countervailing Measures (Subsidies Agreement) of the World Trade
Organization (WTO) provides rules for the use of government subsidies and for the application of
remedies to address subsidized trade that has harmful commercial effects. These remedies can be
pursued through the WTO's dispute settlement procedures or through a countervailing duty (CVD)
investigation, which can be undertaken unilaterally by any WTO member government.
➢ The Subsidies Agreement established a Committee on Subsidies and Countervailing Measures
(WTO Subsidies Committee). The WTO Subsidies Committee meets not less than twice a year.
It allows members to consult on any matters relating to the operation of the Subsidies
Agreement and the advancement of its objectives.

(53-F)
SIA-F-GS I
➢ Thus, The committee was established based on a Committee on Subsidies and Countervailing
Measures (not Agreement on Agriculture )to maintain the list of subsidies allowed by WTO
members. So, Statement 1 is not correct.
WTO Subsidies Committee is composed of representatives of each WTO member country. One way in
which the Subsidies Agreement facilitates compliance with established rules is through subsidy
notifications.
Every WTO member is required to notify the WTO Subsidies Committee each year of any subsidy (as
defined by the Subsidies Agreement) that it is granting or maintaining within its territory. So,
Statement 2 is correct.

72. "Early Harvest Pact," which was seen recently in the news, is related to
(a) Genetically modified short term variety seeds
(b) Advancement of climate goals under the Paris deal
(c) Bilateral trade between two countries on restricted goods
(d) Agreement with pharmacy companies for early access to Vaccines
EXPLANATION:
Early harvest agreements are used to open up bilateral trade between two countries on a restricted
list of goods and services, primarily as a frontrunner to clinching a more comprehensive FTA.
Wrapping up the mode of broad-based FTAs could potentially lead to impediments.
➢ India concluded an early harvest agreement with Thailand in 2004 but has not been able to
conclude a comprehensive FTA with the country. India also has a trade agreement with Sri
Lanka dealing with goods but could not conclude an agreement on services and investments.
➢ Early harvest agreements can be challenged in WTO, as only comprehensive FTAs are exempt
from WTO rules. Article XXIV.8(b) of GATT exempts only those deals which cover substantially
all the trade between two countries.
So, Option (c) is correct.

73. With reference to trade agreements, consider the following statements


1. Preferential Trade Agreement maintains a negative list of products and services on which free
trade is not allowed
2. Free Trade Agreement contains a positive list of Products and services on which free trade is
allowed without any restrictions
Which of the statements given above is/are not correct
(a) 1 only
(b) 2 only
(c) Both 1 and 2
(d) Neither 1 nor 2
EXPLANATION:

(54-F)
SIA-F-GS I
In a Preferential Trade Agreement, two or more partners give preferential right of entry to certain
products. This is done by reducing duties on an agreed number of tariff lines. Here a positive list is
maintained, i.e., the list of the products to which the two partners have agreed to provide preferential
access. Tariffs may even be reduced to zero for some products, even in a PTA. India signed a PTA with
Afghanistan. So, Statement 1 is not correct.
A free trade agreement is an agreement in which two or more countries agree to provide preferential
trade terms, tariff concession etc., to the partner country. Here a negative list of products and services
is maintained by the negotiating countries on which the terms of FTA are not applicable; hence it is
more comprehensive than a preferential trade agreement. India has negotiated FTA with many
countries, e.g., Sri Lanka and various trading blocs as well, e.g., ASEAN. So, Statement 2 is not
correct.

74. With reference to Trade agreements, consider the following statements


1. In trade negotiation, Comprehensive Economic Partnership Agreement(CEPA) has the widest
coverage than Comprehensive Economic Cooperation Agreement (CECAs)
2. India has signed CEPAs only with South Korea and Japan.
Which of the statements given above is/are correct
(a) 1 only
(b) 2 only
(c) Both 1 and 2
(d) Neither 1 nor 2
EXPLANATION:
A partnership agreement or cooperation agreement is more comprehensive than a Free trade
agreement. The comprehensive Economic Cooperation Agreement(CECA) /Comprehensive Economic
Partnership Agreement(CEPA) also looks into the regulatory aspect of trade and encompasses an
agreement covering the regulatory issues.
The CECA has the widest coverage. CEPA covers negotiation on the trade in services and investment
and other areas of economic partnership. It may even consider negotiation in areas such as trade
facilitation and customs cooperation, competition, and IPR. So, Statement 1 is not correct.
Comprehensive Economic Partnership Agreement is more comprehensive and ambitious than a Free
trade agreement (FTA) in terms of partnership across a wider coverage of areas and the type of
commitments. While a traditional FTA focuses mainly on goods, a CEPA is more ambitious in terms
of a holistic coverage of many areas like services, investment, IPR, government procurement, disputes
etc. Secondly, CEPA looks deeper at the regulatory aspects of trade than an FTA.
➢ India has signed CEPAs with South Korea, Japan and UAE.
➢ The India-UAE Comprehensive Economic Partnership Agreement (CEPA) was signed between
the two nations on 18 February 2022.
So, Statement 2 is not correct.

75. The recently adopted Dharmashala declaration is related to


(a) Wildlife protection
(55-F)
SIA-F-GS I
(b) Removing trade barriers among SAARC countries
(c) Tourism sector
(d) Curbing malaria by the end of 2030 in the ASEAN countries
EXPLANATION:
On the occasion of World Tourism Day on September 27, The 'Dharamshala Declaration 2022' was
released at the end of the three-day national conference on tourism attended by ministers from 12
states, besides Governors, UT administrators and central government officials. The Dharamshala
Declaration aims to recognize India's role in contributing towards global tourism as well as focusing
on recovery by also promoting domestic tourism. So, Option (c) is correct.

76. Consider the following statements


1. Vlamudir Putin established the Eastern Economic Forum in 2015 to expand Russia's economic
cooperation in Asia-Pacific.
2. The investors enjoy special customs, tax and administrative regulations in the Free Port
Vladivostok.
Which of the statements given above is/are correct?
(a) 1 only
(b) 2 only
(c) Both 1 and 2
(d) Neither 1 nor 2
EXPLANATION:
The Eastern Economic Forum was established by the President of the Russian Federation, Vladimir
Putin, in 2015 to support the economic development of Russia's Far East and expand international
cooperation in the Asia-Pacific region.
The Eastern Economic Forum is a key international platform for establishing and strengthening ties
within the Russian and global investment communities and for comprehensive expert evaluation of
the economic potential of the Russian Far East, the investment opportunities it offers, and business
conditions within advanced special economic zones. So, Statement 1 is correct.
The Free Port Vladivostok (FPV) is a territory where investors enjoy special customs, tax and
administrative regulations. Within the FPV, tax incentives and administrative preferences apply to
residents. The resident connects to the required infrastructure. The free port system operates in 5
regions of the Far Eastern Federal District: Primorye Territory, Khabarovsky Krai, Kamchatka Region,
Sakhalin Region, and Chukotka Autonomous Area. So, Statement 2 is correct.

77. Consider the following statements about the Child Welfare Committees
1. The state government forms it through official Gazette notification for the care and protection of
children.
2. It has the final authority to dispose of cases related to protecting, developing and rehabilitating
children and human rights issues.

(56-F)
SIA-F-GS I
3. Only a police officer and a public servant can produce the children needing care and protection
before this Committee.

Which of the statements given above is/are correct?


(a) 1 only
(b) 2 and 3 only
(c) 1 and 2 only
(d) 1, 2 and 3
EXPLANATION:
A Child Welfare Committee appointed by the State government under the Juvenile Justice (Care and
Protection of Children) Act 2015, has the authority to dispose of cases of children in need of care and
protection brought before them. It can also order an inquiry to ensure their safety and well-being and
give an order for their rehabilitation either in family-based care such as through restoration to family
or guardian, adoption, foster care or sending them to child care institutions. So, Statement 1 is
correct.
The District Magistrate is the grievances redressal authority for the Child Welfare Committee. Anyone
connected with a child can file a petition before the District Magistrate, who is required to consider
and pass appropriate orders.
The Committee has the authority to dispose of cases for the care, protection, treatment, development
and rehabilitation of children in need of care and protection and to provide for their basic needs and
protection.
As per section-31 of the Juvenile Justice (Care & Protection of Children) Act, the Committee shall have
the final authority to dispose of cases for the care, protection, treatment, development and
rehabilitation of the children as well as to provide for their basic needs and protection of human rights.
So, Statement 2 is correct.
Anybody can produce a child before CWC, including; -
➢ Any police officer or special juvenile police unit or a designated Child Welfare Police Officer;
➢ Any officer of the District Child Protection Unit or an inspector appointed under any labour
law;
➢ Any public servant;
➢ Childline Services;
➢ Voluntary or non-governmental organisation;
➢ Child Welfare Officer or probation officer;
➢ Any social worker;
➢ Public spirited citizen;
➢ Child himself;
➢ Any nurse, doctor or management of a nursing home, hospital or maternity home.
So, Statement 3 is not correct.

(57-F)
SIA-F-GS I
78. Consider the following statements :
1. The Central Bureau of Investigation launched Operation Garuda against internationally linked
drug networks.
2. Odisha is the first State in India to publish tribal communities encyclopedia, including the
particularly vulnerable tribal groups.
3. The world's first Compressed Natural Gas terminal will be set up in Gujarat.
Which of the statements given above is/are correct ?
(a) 1 only
(b) 2 and 3 only
(c) 1 and 2 only
(d) 1, 2 and 3
EXPLANATION:
The Central Bureau of Investigation (CBI) has launched a multi-phase Operation GARUDA to
dismantle drug networks with international linkages. The CBI is doing this in close coordination with
enforcement actions across international jurisdictions through INTERPOL and Narcotics Control
Bureau, with a special focus on Indian Ocean Region. Operation GARUDA seeks to target drug
networks with international footprints for action against handlers, operatives, production zones and
support elements. So, Statement 1 is correct.
Odisha became the first State in the country to start an encyclopedia on tribal communities to
document their age-old and unique traditions.
Tribes, as per the 2011 census, account for 22.85% of Odisha's total population. Odisha is home to
the third largest tribal population in India, but it is the most diverse indigenous community found in
the country. The State has 62 tribes, including 13 particularly vulnerable tribal groups. Five edited
volumes of 'Encyclopedia of Tribes in Odisha' published by Scheduled Castes and Scheduled Tribes
Research and Training Institute and Odisha State Tribal Museum in the Museum complex were
released by Chief Minister Naveen Patnaik. So, Statement 2 is correct.
Prime Minister Narendra Modi laid the foundation stone for the "world's first CNG (Compressed
Natural Gas) terminal" at Bhavnagar in Gujarat, which is expected to be operational in 2026. The
Bhavnagar project will be developed through a public-private partnership with Padmanabhan Mafatlal
Group and the UK's Foresight Group taking the lead. The CNG terminal will come up at an estimated
cost of Rs 4,000 crore. So, Statement 3 is correct.

79. Consider the following statements about AVGAS 100 LL


1. It is a completely imported product from European countries.
2. Recently, IndianOil became the first oil marketing company to manufacture this special aviation
fuel for unmanned aerial vehicles.
(58-F)
SIA-F-GS I
3. Compared to imported grades, this indigenously developed fuel is a lower-octane fuel with slightly
inferior quality standards.
Which of the statements given above is/are correct?
(a) 1 only
(b) 2 and 3 only
(c) 1 and 2 only
(d) 1, 2 and 3
EXPLANATION:
Minister of State for Civil Aviation and Road Transport and Highways launched AVGAS 100 LL, special
aviation fuel meant for piston engine aircraft and Unmanned Ariel Vehicles. Currently, India is
importing this product from European countries.
The launch event hosted by Indian Oil at Hindan Airforce Station witnessed participation by senior
officials from Indian Airforce, senior officials from MoPNG and MoCA and officials from Flying Training
Organizations (FTOs). AVGAS (aviation gasoline) is a gasoline containing tetra-ethyl lead (TEL), which
gives it a high-octane number. This allows the fuel to be used in engines with higher compression
ratios without detonating, which can damage the structure of the combustion chamber. So, they have
been designed to use fuels with high octane ratings, such as AVGAS 100LL, where 100 is the lean
aviation rating, and LL stands for low lead. So, Statement 1 is correct.
Indian Oil has become the first oil marketing company to produce and market AVGAS 100 LL, a special
aviation fuel meant for piston engine aircraft and unmanned aerial vehicles. The domestic production
of AVGAS 100 LL produced by Indian Oil at its Gujarat Refinery will make flying training more
affordable in India. This product which fuels the aircraft operated by FTOs and Defense forces, has
been imported for decades by India. Indian Oil's R&D, Refineries and Marketing teams have achieved
this feat of indigenous production and have offered price advantage to the industry. So, Statement 2
is correct.
Indian Oil is proud to introduce this specialized fuel by leveraging our refining strength and in-house
expertise. In fact, indigenous fuel is superior compared to imported grades. AV Gas market is expected
to grow from the current $ 1.92 billion to $ 2.71 billion by 2029. Indian Oil plan to set up a new
facility soon to target export opportunities, besides catering to the domestic demand. So, Statement
3 is correct.

80. Consider the following statements


1. All mobile phones have a 15-digit unique IMEI number, a unique ID of that particular phone
device.
2. Users can only block their stolen device and cannot unblock the found device on the Indian
Counterfeited Device Restriction portal launched by the Department of Telecommunications.
3. IMEI certificates have now become mandatory, even for imported mobile devices.
Which of the statements given above is/are correct?
(a) 1 and 3 only
(b) 2 and 3 only

(59-F)
SIA-F-GS I
(c) 1 and 2 only
(d) 1, 2 and 3

EXPLANATION:
IMEI stands for International Mobile Equipment Identity. It is a unique 15-digit number and is used
to identify a device on a mobile network. If you have a dual SIM phone, you will have two IMEI numbers
– one for each SIM slot. IMEI numbers are assigned to every GSM phone, while CDMA devices have a
MEID number. So, Statement 1 is correct.
The government introduced the Indian Counterfeited Device Restriction in 2021 for the issue of IMEI
certificates for the import of mobile devices through various customs ports.
The Department of Telecommunications' Central Equipment Identity Register initiative will enable the
government to block stolen or lost devices. If stolen devices start getting blocked, the thieves lose their
interest. However, the central area of concern for the thieves is that the common people will now be
able to track stolen devices through direct assistance from the central government.
The user should block the IMEI of their phone if it has been lost/stolen. The user should unblock the
IMEI of their phone only if it has been found and is in possession of the user. To unblock a lost/stolen
phone's IMEI, the user has to report to the local Police that it is found. After that user can unblock
the phone. So, Statement 2 is not correct.
The international mobile equipment identity number of the mobile phone imported into India for sale,
testing, research or any other purpose shall be registered by the importer with the Indian
Counterfeited Device Restriction portal (https://icdr.ceir.gov.in) of the Government of India in the
Department of Telecommunications prior to import of mobile phone into the country. So, Statement
3 is correct.

81. With reference to silphion, consider the following statements


1. It is a Meditteranean medicinal plant that vanished around 2000 years ago.
2. Though it cured various ailments such as goiter, hormonal disorders, contraceptives and tetanus,
it is not an edible plant.
Which of the statements given above is/are correct?
(a) 1 only
(b) 2 only
(c) Both 1 and 2
(d) Neither 1 nor 2
EXPLANATION:
A Mediterranean medicinal plant considered a cure-all that mysteriously vanished 2,000 years ago
may still be around, a recent study claimed. The resin of the silphion was extensively used as a spice,
perfume, aphrodisiac, contraceptive and medicine. It occupied an important place in the export

(60-F)
SIA-F-GS I
economy of ancient Cyrene, an old Greek and later Roman colony near north-eastern Libya, according
to a 2022 paper published in the journal Heritage. So, Statement 1 is correct.
Silphion was used to treat various health problems, including goiter, sciatica (nerve pain), toothache,
intestinal disorders, hormonal disorders, epilepsy, tetanus, polyps (abnormal growth of tissues) and
malignant tumors, according to studies.
Its stalks were(edible plant) eaten as a vegetable, while the roots were consumed raw. The plant was
also used to preserve lentils, historical records showed. So, Statement 2 is not correct.

82. Consider the following statements about the Alternate Investment Funds (AIF)
1. Any private investment fund, irrespective of Indian or foreign sources, is termed AIF.
2. It can be in the form of trust, company, body corporate or limited liability partnership.
3. Some AIFs are regulated by RBI and PFRDA also.
Which of the statements given above is/are correct?
(a) 1 and 3 only
(b) 2 and 3 only
(c) 1 and 2 only
(d) 1, 2 and 3
EXPLANATION:
In India, alternative investment funds (AIFs) are defined in Regulation 2(1) (b) of the Securities and
Exchange Board of India (Alternative Investment Funds) Regulations, 2012. Alternative Investment
Fund or AIF means any fund established or incorporated in India which is a privately pooled
investment vehicle that collects funds from sophisticated investors, whether Indian or foreign, for
investing it in accordance with a defined investment policy for the benefit of its investors. So,
Statement 1 is correct.
An AIF, under the SEBI (Alternative Investment Funds) Regulations, 2012 can be established or
incorporated in the form of a trust or a company or a limited liability partnership or a body corporate.
Most of the AIFs registered with SEBI are in trust form. So, Statement 2 is correct.
Hence, in India, AIFs are private funds that are otherwise not coming under the jurisdiction of any
regulatory agency in India. So, Statement 3 is not correct.

83. Madhav national park is located in


(a) Maharashtra
(b) Madhya Pradesh
(c) Uttar Pradesh
(d) Karnataka
EXPLANATION:
The Madhav National Park is situated on the northern fringe of the Central Highlands of India, forming
a part of the Upper Vindhyan Hills intermixed with plateaus and valley sections. The slopes are

(61-F)
SIA-F-GS I
generally gentle and rarely steep. Spreading over an area of almost 355 sq km, Madhav National Park
is a fascinating mix of natural splendors of history and architectural wonders. Madhav National Park
is situated in the northern part of Madhya Pradesh in Shivpuri District, very close to the Shivpuri
township.
This was notified as a National Park in the year 1958.
➢ Flora and Fauna: The Park represents the Northern Tropical dry deciduous mixed forest as
well as dry thorn forest, typical of North-Western Madhya Pradesh. Kardhai is the dominant
tree species. The forests here are home to antelopes like Nilgai, Chinkara and Chowsinga and
Deers including Chital, Sambar and Barking Deer. One can see animals like Leopard, Wolf,
Jackal, Fox, Wild Dog, Wild Pig, and Porcupine. Marsh Crocodile, Turtles and Snakes
represent the reptilian fauna
So, Option (b) is correct.

84. Consider the following statements


1. India's first avalanche monitoring radar was deployed in Sikkim by the Indian Army.
2. The radar mentioned above can detect avalanches within ten minutes of their trigger in high-
altitude areas.
Which of the statements given above is/are correct?
(a) 1 only
(b) 2 only
(62-F)
SIA-F-GS I
(c) Both 1 and 2
(d) Neither 1 nor 2
EXPLANATION:
An avalanche monitoring radar, the first of its kind in India, has been installed in North Sikkim by
the Army and Defence Geoinformatics and Research Establishment (DGRE). It has the capability to
detect avalanches within three seconds of their trigger and will assist in saving the lives of troops and
reducing damage to property in super high altitude areas. The radar was inaugurated by Lt. Gen.
Tarun Kumar Aich, General Officer Commanding of Sukna-based 33 Corps, on September 20 at one
of the forward posts of the Army at an altitude of 15,000 feet in North Sikkim. So, Statement 1 is
correct.
The radar uses a series of short microwave pulses, which are scattered at the target and can detect
an avalanche in under three seconds, not three minutes. "The radar can permanently scan the
targeted slope for avalanche release and track the path of the avalanche and its size in case it is
triggered; the radar can see through snow, fog, and at night, making it an all-weather solution. It
covers an area of two sq. km, obviating the requirement to place additional instruments in dangerous
avalanche-prone areas. The radar is also linked to an alarm system, enabling automatic control and
warning measures in case an avalanche is triggered. So, Statement 2 is not correct.

85. Consider the following statements about the river Thamiraparani


1. It is the perennial river in South India.
2. It originates and ends in Tamil Nadu state itself.
3. It has been mentioned in the Sangam literature.
Which of the statements given above is/are correct?
(a) 1 only
(b) 2 and 3 only

(63-F)
SIA-F-GS I
(c) 1 and 2 only
(d) 1, 2 and 3
EXPLANATION:
The Thamiraparani is unique in many respects. It is the only perennial river in Tamil Nadu. It
originates in the Pothigai Hills of the Western Ghats in Tirunelveli district.
The district administration of Tirunelveli in Tamil Nadu along with Bengaluru-based non-profit
Ashoka Trust for Research in Ecology and the Environment (ATREE) are using what they term as a
'hyper local' approach to restore one of south Asia's oldest rivers, the Thamirabarani. So, Statement
1 is correct.
The river flows through Tirunelveli and then neighbouring Thoothukudi and ends in the Gulf of
Mannar at Punakayil. It thus originates and ends in the same State. The river supports wildlife such
as the Nilgiri marten, slender loris, lion-tailed macaque, white spotted bush frog, galaxy frog, Sri
Lankan Atlas moth and the great hornbill. The TamiraSES project aims to restore the Social-Ecological
Systems of the Tamiraparani river riverscape from head-waters to the estuary to enable conditions for
native biodiversity to thrive and maintain and enhance multiple ecosystem services to local
stakeholders. So, Statement 2 is correct.
Besides the ecosystem services it provides, the river also has historical value for the people of the
State. It is mentioned extensively in Sangam Era literature. So, Statement 3 is correct.

86. Consider the following statements about Swachh Toycathon


1. Toys will be manufactured based on the circular economy concept.
2. Ministry of Housing and Urban Affairs, in collaboration with IIT Gandhinagar, has launched it.
Which of the statements given above is/are correct?
(a) 1 only
(b) 2 only
(64-F)
SIA-F-GS I
(c) Both 1 and 2
(d) Neither 1 nor 2
EXPLANATION:
National Action Plan for Toys (NAPT) 2020 was introduced to promote the Indian toy industry,
including traditional handicrafts and handmade toys with the objective of establishing India as a
global Toy hub. The Ministry of Housing and Urban Affairs (MoHUA) launched Swachh Toycathon
under the Swachh Amrit Mahotsav aims to explore solutions for the use of waste in the creation or
manufacturing of toys.
The Swachh Toycathon is a national competition for individuals and groups. It is based on three broad
themes (i) fun and learns seeks ideas for design and an early prototype of toys from waste at home,
workplace and surroundings, (ii) use and enjoy ideas for design and models of games and play in the
park/open spaces made from waste and (iii) new and old that seeks ideas/solutions/working models
for circularity in the toy industry. So, Statement 1 is correct.
Swachh Toycathon is a competition being undertaken by the Ministry of Housing and Urban Affairs
under Swachh Bharat Mission-Urban (SBM-u 2.0) with the aim of 'Rethinking Indian toy industry'
with Center for Creative Learning, IIT Gandhinagar is the knowledge partner for the initiative. So,
Statement 2 is correct.

87. With reference to flex-fuel technologies and vehicles, consider the following statements
1. These vehicles can run on petrol and ethanol blending up to 100%.
2. Currently, these vehicles are available in US, Brazil and Canada.
3. Recently, in India, a flex-fuel car was launched by Toyota.
Which of the statements given above is/are correct?
(a) 1 only
(b) 2 and 3 only
(c) 1 and 2 only
(d) 1, 2 and 3
EXPLANATION:
Flex fuel vehicles can run on flexible fuels- petrol, ethanol or a blend of petrol and ethanol. Flex Fuel
Vehicles are not confined to a certain kind of fuel and can flexibly run on a blend of petrol and ethanol
up to 83%. Such vehicles offer the flexibility to switch to engine fuel from petrol to ethanol. They can
also power the engine from the battery as well, thus helping to reduce carbon emissions from it. As of
now, ethanol is way cheaper than petrol in India, providing scope for flex-fuel car owners to use the
fuel to save on their fuel bills. It will also help India in reducing its dependence on fuel imports. India
has advanced the target date for achieving 20% ethanol-blending in petrol by five years to 2025. So,
Statement 1 is not correct.
As of now, Flex Fuel Vehicles are available in USA, Brazil, and Canada. According to an official US
website, as of 2018, around 21 million Flexi Fuel Vehicles were plying on the roads of the United

(65-F)
SIA-F-GS I
States. According to the US Department of Energy, FFVs are more efficient and show improved
acceleration performance when fuelled with higher ethanol blends. So, Statement 2 is correct.
Toyota Corolla Altis Hybrid is a first-of-its-kind pilot project vehicle that is based on Flexi-Fuel Strong
Hybrid Electric Vehicles (FFV-SHEV) technology. It is powered by a 1.8-liter ethanol-ready petrol-
hybrid engine. The initiative will power India's aim to be among the top global producers in every
segment of vehicles over the next 25 years, as stated by the Society of Indian Automobile
Manufacturers. So, Statement 3 is correct.

88. Rosh Hashanah is associated with which community?


(a) Jains
(b) Buddhists
(c) Jews
(d) Muslims
EXPLANATION:
Jews throughout the globe celebrate the Jewish New Year, popularly called Rosh
Hashanah. Rosh Hashanah is considered one of the holiest days in Judaism, Tishrei
almost always coincides with September or October every year. Rosh Hashanah is the
Jewish New Year celebrated on the first day of Tishrei, the seventh month of the Hebrew
calendar. So, Option (c) is correct.

89. Which countries recently signed the Humanitarian Assistance and Disaster Relief (HADR)
partnership?
1. India

(66-F)
SIA-F-GS I
2. Russia
3. United Kingdom
4. Australia
5. Japan
6. United States of America
7. Bhutan
8. Nepal
Select the correct answer using the code given below
(a) 1, 2, 3, 4 and 5 only
(b) 1, 3, 4, 5 and 6 only
(c) 1, 4, 7 and 8 only
(d) 1, 4, 5 and 6 only
EXPLANATION:

The HADR mechanism was announced by Quad leaders in Tokyo in May. The partnership has been
announced as part of a shared vision for a "free and open Indo-Pacific that is inclusive and resilient,"
the US State Department said in a press release. The signing of these guidelines demonstrates a
significant moment in Quad's cooperation.
The leaders from the Quad countries signed the Humanitarian Assistance and Disaster Relief (HADR)
partnership guidelines. The 'Quadrilateral Security Dialogue' (QSD) is an informal strategic forum
comprising four nations, namely -- United States of America (USA), India, Australia and Japan. The
forum committed to bringing tangible benefits to the region and ensuring a peaceful, stable,
prosperous region where sovereignty is respected, and where the countries are free to make their own
choices. So, Option (d) is correct.

90. Consider the following statements about the Tomistoma


1. It is a shy species that usually retreat from human activity areas.
2. It does not share its habitat with other crocodile species anywhere on the earth.
3. Recently, due to illegal gold mining and oil palm plantations, this Tomistoma species has been
attacking humans in Indonesia.
Which of the statements given above is/are correct?
(a) 1 only
(b) 2 and 3 only
(c) 1 and 3 only
(d) 1, 2 and 3
EXPLANATION:
The Tomistoma (Tomistoma schlegelii) is a shy and reclusive species that typically retreat from areas
with high human activity. However, increasing human intrusion into its habitat may be causing it to

(67-F)
SIA-F-GS I
attack people, a new study has suggested. The Tomistoma is a large, slender-snouted crocodilian
species native to Southeast Asia. It is distributed across parts of Borneo (divided between Indonesia,
Malaysia and Brunei), peninsular Malaysia and Sumatra. It is a freshwater species and is frequently
associated with peat swamp forests. It is currently listed as Endangered on the International Union for
the Conservation of Nature Red List of Threatened Species So, Statement 1 is correct.
The Tomistoma shares its habitat with two other types of crocodilians — the saltwater crocodile and
the Siamese crocodile. Report of crocodilian attacks in Indonesia using media reports and
communication with local authorities. They analyzed the data to confirm whether the species involved
in the attack was a Tomistoma. So, Statement 2 is not correct.
They blame human activities such as oil palm plantations, settlements and illegal gold mining for the
rise in attacks. The researchers also noted that at least one research in the past had talked about
tomistomas becoming accustomed to human presence.
Such behavior was noted in all the attacks that the present study analyzed. Also, nearly half of all
victims of attacks that the study covered were fully submerged in water at the time of the attack. Local
inquiries had also suggested that tomistoma had never attacked boats or people on shore. The authors
of the study collected reports of crocodilian attacks in Indonesia using media reports and
communication with local authorities. So, Statement 3 is correct.

91. Recently, the Prime Minister of India declared which village is India's first 24x7 solar-powered village?
(a) Ziro
(b) Modhera

(68-F)
SIA-F-GS I
(c) Mawlynnong
(d) Zuluk
EXPLANATION:
A 24x7 solar-powered village means that the village's energy needs are met entirely through solar
power, and the residents have access to reliable and uninterrupted electricity.Prime Minister of India
declared the village of Modhera in Gujarat as India’s first 24x7 solar-powered village.
It is first-of-its-kind project that realises the vision of solarisation of the sun-temple town of Modhera.
It involved developing a Ground Mounted Solar power plant and Rooftop solar systems on residential
& Government buildings, all integrated with Battery Energy storage system (BESS).
The project will demonstrate how India’s renewable energy prowess can empower people at the
grassroots. So, Option b is correct.

92. Consider the following statements about Athlete's Biological Passport (ABP)
1. It is an anti-doping tool that enhances testing, analysis, investigations and deterrence.
2. It will detect the doping substance.
Which of the statements given above is/are correct?
(a) 1 only
(b) 2 only
(c) Both 1 and 2
(d) Neither 1 nor 2
EXPLANATION:
The Athlete Biological Passport (ABP) an anti-doping tool by the World Anti-Doping Agency (WADA) to
monitor and detect the use of performance-enhancing drugs (PEDs) by athletes. Athlete Biological
Passport (ABP) works against doping through enhanced target testing and analysis, investigations,
deterrence, and as indirect evidence for use of prohibited methods or substances. So, Statement 1
is correct.
Athlete Biological Passport (ABP), a powerful anti-doping tool, although it cannot detect the doping
substance or method itself, ABP monitors selected biological variables over time to reveal the effects
of doping. So Statement 2 is not correct.

93. Consider the following statements about banknotes


1. The sole authority to issue banknotes in India is the Reserve Bank of India.
2. Though RBI issues banknotes, the PMO office changes the design of banknotes.
3. Banknotes with Mahatma Gandhi series was introduced in India in 1996 and has been in
circulation in the economy since then.
Which of the statements given above is/are correct?
(a) 2 only
(b) 1, 2 and 3
(c) 2 and 3 only
(69-F)
SIA-F-GS I
(d) 1 and 3 only

EXPLANATION:
The RBI was established on April 1, 1935, as the central bank of India. The responsibility of issuing
currency notes in India is vested with the RBI under the Reserve Bank of India Act, 1934.Thus sole
authority to issue banknotes in India So, Statement 1 is correct.
Reserve Bank, like other central banks worldwide, occasionally changes the design of banknotes with
enhanced security features to stay ahead of counterfeiters. In addition to security and durability, the
RBI may also change the design of banknotes to reflect the cultural and historical heritage of the
country. It is RBI which changes the Bank note design NOT PMO office.
So, Statement 2 is not correct.
The Mahatma Gandhi Series of banknotes replaced the earlier Lion Capital Series of banknotes, which
had been in circulation since 1957. Mahatma Gandhi Series has been in circulation in the economy
since then. The Mahatma Gandhi Series of banknotes is currently in its fourth iteration, with the
latest version having been introduced in 2018. The latest version of the series features several new
security features, including a fluorescent stripe, intaglio printing, and a see-through register. So,
Statement 3 is correct.

94. Consider the following statements about Chhath Puja


1. This festival is dedicated to the Sun God, mainly celebrated in Bihar, Uttar Pradesh, Jharkhand,
and the Madhesh region of Nepal.
2. It is mentioned in the Puranas of Ramayana and Mahabharata.
3. This festival leads to a disturbance of nature and the environment since people pollute the areas
of worship.
Which of the statements given above is/are correct?
(a) 2 only
(b) 1, 2 and 3
(c) 2 and 3 only
(d) 1 and 2 only
EXPLANATION:
The auspicious festival of Chhath Puja is dedicated to Lord Surya (Sun God) and his sister Shashthi
Devi (Chhathi Maiya) mainly celebrated in Bihar, Uttar Pradesh, Jharkhand, and the Madhesh region
of Nepal. So, Statement 1 is correct.
Chhath is mentioned in both Ramayana and Mahabharata, which are two of the most important
Hindu epics. In Ramayana, Goddess Sita did the Chhath puja on the day of the establishment of Ram-
Rajya (the kingdom of Lord Ram). In Mahabharata, it was performed by Pandava-mother Kunti after
they managed to escape from Lakshagrih, the palace made of lacquer, which was burnt to the ground.
So, Statement 2 is correct.

(70-F)
SIA-F-GS I
95. Consider the following statements
1. Flexi-Fuel Strong Hybrid Electric Vehicles (FFV-SHEV) will run on 100 % petrol, blended ethanol
and electric power.
2. The flex-fuel vehicles have improved performance when operating on higher ethanol blends.
3. Absolute alcohol is intended for human consumption.
Which of the statements given above is/are correct?
(a) 2 only
(b) 1, 2 and 3
(c) 2 and 3 only
(d) 1 and 2 only
EXPLANATION:
Flexi-Fuel Strong Hybrid Electric Vehicles (FFSHVs) are vehicles that use both a flexible fuel engine
and an electric motor for propulsion. Flexi-Fuel Strong Hybrid Electric Vehicles (FFV-SHEV) in India
would run on 100% petrol as well as 20 to 100% blended ethanol and electric power. So, Statement
1 is correct.
The ethanol blends used in FFSHEVs are typically E85, which is a mixture of 85% ethanol and 15%
gasoline. Ethanol has a higher octane rating than gasoline, which means that it can improve the
performance of the engine while reducing emissions. So, Statement 2 is correct.
Absolute ethanol or anhydrous alcohol is commonly used in laboratory settings as a solvent, a
disinfectant, and a fuel. However, it is highly flammable and should be handled with caution. Absolute
ethanol generally refers to purified ethanol containing no more than one percent water. Absolute
alcohol is not intended for human consumption .So, Statement 3 is not correct.

96. Consider the following statements about LVM3


1. Launch Vehicle Mark 3 (LVM3) M2, India's heaviest rocket, has placed 36 satellites of OneWeb in
orbit.
2. The OneWeb constellation satellites will operate in geosynchronous orbit, and each satellite
completes a round trip around the earth in 109 minutes.
Which of the statements given above is/are correct?
(a) 1 only
(b) 2 only
(c) Both 1 and 2
(d) Neither 1 nor 2
EXPLANATION:
Indian Space Research Organisation (ISRO) launched its first dedicated commercial mission of Launch
Vehicle Mark 3 (LVM3) M2, India's heaviest rocket in October 2022. It placed 36 satellites of UK based

(71-F)
SIA-F-GS I
One Web in orbits. The Launch Vehicle Mark-3 (LVM 3),[1][13] previously referred as the
Geosynchronous Satellite Launch Vehicle Mark III (GSLV Mk3), is a three-stage medium-lift launch
vehicle developed by the Indian Space Research Organisation (ISRO).
So, Statement 1 is correct.
The one web constellation be arranged in 12 rings with 49 satellites in each plane to build up a
constellation. Each satellite will complete a full trip around the earth every 109 minutes. The mission
will enhance telecommunication and related services. Constellation will operate in an LEO polar orbit
NOT in geosynchronous orbit. So, Statement 2 is not correct.

97. Consider the following statements about Mars Orbiter Mission


1. It is India's first interplanetary mission to Mars, designed for 6 months of life.
2. Methane Sensor for Mars measures the concentration of methane in the Martian atmosphere.
3. This mission observed one of the moons of Mars known as Deimos.
Which of the statements given above is/are correct?
(a) 2 only
(b) 2 and 3 only
(c) 1 and 2 only
(d) 1, 2 and 3
EXPLANATION:
The Mars Orbiter Mission (MOM), also known as Mangalyaan, is a space mission launched by the
Indian Space Research Organisation (ISRO) in November 2013. It is India's first interplanetary mission
to Mars. Though the designed mission life is 6 months, MOM completed 7 years in its orbit on Sept
24, 2021.So, Statement 1 is correct.
Mars Orbiter Mission carried the following five scientific payloads:
1. Mars Color Camera (MCC)
2. Thermal Infrared Imaging Spectrometer (TIS)
3. Methane Sensor for Mars (MSM)
4. Mars Exospheric Neutral Composition Analyser (MENCA)
5. Lyman Alpha Photometer (LAP).
The Primary science objective of Methane Sensor for Mars is to detect and measure methane
concentration in Martian atmosphere (if it does exist) under clear sky conditions.
So, Statement 2 is correct.
Uniqueness of MOM:
➢ Highly elliptical orbit geometry of MOM enables its Camera (MCC) to take snap shots of Full
disc of Mars at its farthest point and finer details from closest point.
➢ First time observation of the far side of Deimos, one of the moons of Mars
So, Statement 3 is correct

(72-F)
SIA-F-GS I
98. Recently, Japan has sought a Geographical Indication tag for nihonshu. It is related to
(a) Textile craft
(b) Wheat variety
(c) Alcoholic beverage
(d) Sweet prepared from jaggery and sesame
EXPLANATION:
Geographical Indication (GI) is a type of intellectual property that identifies a product as originating
from a specific geographic location, where the quality, reputation, or other characteristic of the
product is essentially attributable to its geographical origin.
Geographical Indication is governed and directed by the WTO Agreement on Trade-Related Aspects of
Intellectual Property Rights (TRIPS).
In Japan, Nihonshu is regarded as a special and valuable Alcoholic beverage made from
fermenting rice. Japan has sought a Geographical Indication tag for nihonshu .So, Option © is
correct.

99. The One Health Joint Plan of Action was launched by


1. Food and Agriculture Organization of the United Nations (FAO),
2. United Nations Environment Programme (UNEP)
3. World Health Organization (WHO)
4. World Organisation for Animal Health (WOAH, founded as OIE)
Select the correct answer using the code given below
(a) 2, 3 and 4 only
(b) 1, 2 and 3 only
(c) 1, 3 and 4 only
(d) 1, 2, 3 and 4
EXPLANATION:
One Health Joint Plan of Action aims to create a framework to integrate systems and capacity so that
we can collectively better prevent, predict, detect, and respond to health threats. Ultimately, this
initiative seeks to improve the health of humans, animals, plants, and the environment, while
contributing to sustainable development.
The One Health Joint Plan of Action was launched by the Quadripartite –
• Food and Agriculture Organization of the United Nations (FAO),
• United Nations Environment Programme (UNEP),
• World Health Organization (WHO), and
• World Organisation for Animal Health (WOAH, founded as OIE).
So, Option (d) is correct.

(73-F)
SIA-F-GS I
100. Consider the following statements
1. The mission schools of excellence will enhance the infrastructure development of schools by
building new classrooms, smart classrooms and computer labs.
2. Lothal In Gujarat is a trading center of Indus Valley Civilisation and India's maritime power and
prosperity.
Which of the statements given above is/are correct?
(a) 1 only
(b) 2 only
(c) Both 1 and 2
(d) Neither 1 nor 2
EXPLANATION:
The Prime Minister, launched Mission Schools of Excellence at Trimandir, Adalaj, Gujarat. The
Mission will help strengthen education infrastructure in Gujarat by setting up new classrooms, smart
classrooms, computer labs and overall upgradation of the infrastructure of schools in the State. So,
Statement 1 is correct.
Lothal was not only a major trading centre of the Indus Valley Civilization, but it was also a symbol
of India's maritime power and prosperity, Lothal was situated on the banks of the Sabarmati River
and was an important port city, serving as a gateway for trade between the Indus Valley Civilisation
and the rest of the world. So, Statement 2 is correct.

(74-F)
SIA-F-GS I

You might also like